Вы находитесь на странице: 1из 44

!"#$%&"'( * &++,- .$!

/#0" 1
Anbochi, Atillo, Weigand
Taken from Rhys Alexeis reviewer. Digests from upperbatch.

Sections 1-25
Constantino v Asialife
This case involves a consolidation of 2 cases:
Case No. 1: For the sum of 175.04 as annual premium duly paid to ALIC, it
issued Policy No. 93912 whereby it insured the life of Arcadio Constantino for
20 years for P3T with Paz Constantino as beneficiary. First premium covered
the period up to Sept. 26, 1942. No further premiums were paid after the first
premium and Arcadio died on Sept. 22, 1944. Due to Jap occupation, ALIC
closed its branch office in Manila from Jan. 2 1942-1945.

Case No. 2: For the sum of P3T for 20 years. The annual premium stipulated
was regularly paid from Aug. 1, 1938 up to and including Sept. 30, 1940.
Effective Aug. 1, 1941, the mode of payment was changed from annually to
quarterly and such quarterly premiums were paid until Nov. 18, 1941. Last
payment covered the period until Jan. 31, 1942. Tomas Ruiz died on Feb. 16,
1945 with Agustina Peralta as his beneficiary. Due to Jap occupation, it became
impossible and illegal for the insured to deal with ALIC. Aside from this the
insured borrowed from the policy P234.00 such that the cash surrender value of
the policy was sufficient to maintain the policy in force only up to Sept. 7, 1942.

Both policies contained this provision: All premiums are due in advance and any
unpunctuality in making such payment shall cause this policy to lapse unless
and except as kept in force by the grace period condition.
Paz Constantino and Agustina Peralta claim as beneficiaries, that they are
entitled to receive the proceeds of the policies less all sums due for premiums in
arrears. They also allege that non-payment of the premiums were caused by
the closing of ALICs offices during the war and the impossible circumstances
by the war, therefore, they should be excused and the policies should not be
forfeited.
Lower court ruled in favor of ALIC.

Issue: May a beneficiary in a life insurance policy recover the amount thereof
although the insured died after repeatedly failing to pay the stipulated
premiums, such failure being caused by war?
Held: NO. Due to the express terms of the policy, non-payment of the premium
produces its avoidance. In Glaraga v. Sun Life, it was held that a life policy was
avoided because the premium had not been paid within the time fixed; since by
its express terms, non-payment of any premium when due or within the 31

day
grace period ipso fact caused the policy to lapse.
When the life insurance policy provides that non-payment of premiums will
cause its forfeiture, war does NOT excuse non-payment and does not avoid
forfeiture. Essentially, the reason why punctual payments are important is that
the insurer calculates on the basis of the prompt payments. Otherwise,
malulugi sila.
It should be noted that the parties contracted not only as to peace time
conditions but also as to war-time conditions since the policies contained
provisions applicable expressly to wartime days. The logical inference therefore
is that the parties contemplated the uninterrupted operation of the contract even
if armed conflict should ensue.
Insular Life v Ebrado
Buenaventura Ebrado was issued by Insular Life Assurance Co. a whole life
plan for P5,882.00 with a rider for Accidental Death Benefits for the same
amount.
Ebrado designated Carponia Ebrado as the revocable beneficiary in his policy,
referring to her as his wife.
Ebrado died when he was accidentally hit by a falling branch of tree.
Insurer by virtue of the contract was liable for 11,745.73, and Carponia filed her
claim, although she admitted that she and the insured were merely living as
husband and wife without the benefit of marriage.
Pascuala Ebrado also filed her claim as the widow of the deceased insured.
Insular life filed an interpleader case and the lower court found in favor of
Pascuala.

Issue: Between Carponia and Pascuala, who is entitled to the proceeds?
Held: The proceeds should go to the estate of the deceased insured. It is
quite unfortunate that the Insurance Act or our own Insurance Code does not
contain a specific provision grossly resolutory of the prime question at hand.
Rather, the general rules of civil law should be applied to resolve this void in the
insurance law. Art. 2011 of the NCC states: The contract of insurance is
governed by special laws. Matters not expressly provided for in such special
laws shall be regulated by this Code. When not otherwise specifically provided
for in the insurance law, the contract of life insurance is governed by the general
rules of civil law regulating contracts.

!"#$%&"'( * &++,- .$!/#0" 2
Anbochi, Atillo, Weigand
Taken from Rhys Alexeis reviewer. Digests from upperbatch.

Under Art. 2012, NCC: Any person who is forbidden from receiving any
donation under Art. 739 cannot be named beneficiary of a life insurance policy
by a person who cannot make any donation to him, according to said article.
Under Art. 739, donations between persons who were guilty of adultery or
concubinage at the time of the donation shall be void.
In essence, a life insurance policy is no different from civil donations insofar
as the beneficiary is concerned. Both are founded on the same consideration of
liberality. A beneficiary is like a donee because from the premiums of the policy
which the insured pays, the beneficiary will receive the proceeds or profits of
said insurance. As a consequence, the proscription in Art. 739 should equally
operate in life insurance contracts.
Therefore, since common-law spouses are barred from receiving donations,
they are likewise barred from receiving proceeds of a life insurance contract.
However, the wife also cannot receive the proceeds, as if the beneficiary is
disqualified from being one, the proceeds of the policy shall go to the estate of
the deceased insured contracting party.
Qua Chee Can v Law Union & Rock Co.
Qua Chee Gan owned 4 warehouses/bodegas in Albay, used for the storage of
stock of copra, hemp, bale and loose, which he dealt extensively. These
warehouses & their contents were insured by the defendant insurance
company, Law Union & Rock Insurance.

Fire of undertermined origin completely destroyed Bodegas 1, 3, and 4, with the
merchandise stored in them (fire lasted for almost 1 week). Qua Chee informed
the company but the latter resisted payment, claiming violation of warranties &
conditions, filing of fraudulent claims, and that the fire had been deliberately
caused by the insured or by other persons in connivance with him.

Qua Chee Gan, et al. were tried for the crime of arson, it being claimed that
they had set fire to destroy warehouses to collect insurance. They were
acquitted by the CFI. Hence, the civil suit to collect the insurance money. The
CFI ordered the Law Union to pay Qua Chee Gan.

Law Union appealed to the SC, alleging that the policies were avoided for
breach of warranty. Qua Chee Gan should have had 11 fire hydrants as
stipulated, but he only had 2. He also breached the Hemp Warranty provisions
against storage of gasoline, since he admitted that there were 36 cans of
gasoline in Bodega 2.
Issue: W/N the insurance policies were avoided for breach of warranty NO

Ratio:
RE: Hydrant Warranty
The SC agreed with the CFIs ruling that the insurance company is barred by
waiver or estoppel to claim violation of the hydrant warranty. The they knew
very well that there were only 2 installed, but nevertheless issues the policies
and received the corresponding premiums. American jurisprudence holds that
where the insurer, at the time of the issuance of a policy of insurance, has
knowledge of existing facts which, if insisted on, would invalidate the contract
from its very inception, such knowledge constitutes a waiver of conditions in the
contractand the insurer is stopped from asserting the breach of such
conditions. The reason for the rule is human justice. To hold otherwise would
allow the company to treat the policy as valid and receive premiums on it, and
leave it at liberty to repudiate it the next moment.

RE: Hemp Warranty (Related to our topic)
Qua Chee Gan did not violate the Hemp Warranty as well. Gasoline is not
specifically mentioned in the prohibited articles listed. The policy actually spoke
of oils (having a flashpoint below 300 degrees F), which is ambiguous and
uncertain, for in ordinary parlance, this would mean lubricants, not kerosene or
gasoline. And it is a wonder how many insured could understand or determine
flashpoint below 300 degrees.

By reason of the exclusive control of the insurance company over the
terms & and phraseology of the contract, the ambiguity must be held
strictly against the insurer and liberally in favor of the insured.

Insurance is, in its nature, complex & difficult for the layman to understand.
Policies are prepared by experts who know and can anticipate the bearing and
possible complications of every contingency. So long as insurance companies
insist upon the use of ambiguous, intricate & technical provisions which conceal
their own intentions, the courts must, in fairness to those who purchase
insurance, construe every ambiguity in favor of the insured (Algoe v. Pacific
Mut. L. Ins. Co.).

The court sees no reason why the prohibition of keeping gasoline could not be
expressed clearly and unmistakably. The contract of insurance is one of perfect
good faith, not for the insured alone, but equally so for the insurer.

Furthermore, the gasoline kept was incidental to the insureds business. It is a
well-settled rule that keeping of inflammable oils (weird, SC already ruled that
gasoline isnt considered an oil) on the premises, though prohibited by the

!"#$%&"'( * &++,- .$!/#0" 3
Anbochi, Atillo, Weigand
Taken from Rhys Alexeis reviewer. Digests from upperbatch.

policy, does not void it if such keeping is incidental to the business. Also, the
hemp warranty forbade storage only in the building to which the insurance
applies. In this case, no gasoline was stored in the burned bodegas, and
Bodega 2 was isolated from them.

_______________________________________________________________
______
Ty v Filipinas Cia. De Seguros

Petitioner was a mechanic operator for a cotton factory. He obtained a Personal
Accident Insurance Policies from various companies including herein defendant.
The factory caught fire. While he was aiding in the extinguishment of the fire, a
heavy object fell on petitioners left hand causing injuries. Petitioners orthopedic
surgeon certified that his left hand had suffered from temporary total disability.
Petitioner sued defendant over the policy. He contends that it is sufficient that
the injury would result to incapacity to discharge the duties of his work
Defendant contends that, upon the clear terms of the policy, in order for the
injury to be compensable, the hand must be amputated.

HELD: Ty cannot recover on the insurance policies.
The provision in the indemnity agreement is clear and leaves no room for other
construction. To be compensable, the hand must be amputated and not merely
fractured as was the case of petitioner. The insurance policy is the law between
the parties.
_______________________________________________________________
______Philam Life v. Ansaldo

Private respondent Paterno wrote a letter to herein respondent Ansaldo, then
Insurance Commissioner, to complain about problems encountered by agents,
supervisors, managers and public consumers of Philippine American Life
Insurance (Philam Life) as a result of certain practices of the latter. Paterno
caims that there were provisions of the Contract of Agency executed between
Philam Life and its agents that are illegal. Ansaldo requested Delos Reyes,
Philam Lifes President, to comment on Paternos letter. Delos Reyes however
requested for some sort of bill of particulars in order that he can prepare an
intelligent reply. Paterno requested instead that a hearing be conducted.

Paterno wanted that the contracts provisions on charges and fees be declared
null and void, and that charges and fees already collected by Philam Life be
reimbursed to the agents. Delos Reyes on the other hand, submitted an answer
stating that since the Commissions quasi-judicial power was being invoked,
Paterno should file a verified formal complaint before any further proceeding
may take place.

The Commissioner then notified both parties of the hearing of the case. Philam
Lifes Senior AVP filed a motion to quash subpoena/notice, one of the grounds
being that the Insurance Commission has no jurisdiction over both the action
and the parties. The Commissioner denied the motion to quash, ruling that the
case is cognizable by the Commission. Hence this.

Issue: w/n the case (the resolution of the legality/validity of the agency contract)
falls within the jurisdiction of the Commissioner

Ruling: The insurance commissioner does not have jurisdiction over the case.
The general regulatory authority of the Insurance Commissioner is stated in
Sec.414 of the Insurance Code (Commissioner has the duty to see that all laws
relating to insurance... are faithfully executed) and under Sec415
(Commissioner is authorized to impose upon insurance companies fines and
suspensions for violations of the Code). Simply, all this means that the
Commissioner has the authority to regulate the business of insurance,
which is defined to include (a) making or proposing to make as insurer
any insurance contract; (b) making or proposing to make as surety any
contract of suretyship as a vocation and not merely incidental to any
other legitimate business or activity of the surety, (c) doing any kind of
business, including a reinsurance business, specifically recognized as
constituting the doing of an insurance business within the meaning of this
Code, (d) doing or proposing to do any business in substance equivalent
to any of the foregoing in a manner designed to evade the provisions of
this Code.

Since the contract of agency between Philamlife and its agents is not included
within the meaning of an insurance business, then Sec 2 cannot be invoked to
give jurisdiction over the same to the Commissioner. Expressio unius est
exclusion alterius.

Even the quasi-judicial power of the Commissioner as stated in Sec.416 cannot
be invoked because it only refers to claims involving loss, damage or liability for
which an insurer may be answerable. This power does not cover the
relationship affecting the insurance company and its agents. The Code does not
have provisions governing this kind of relationship. Commissioner has no
jurisdiction.


Philamcare vs CA

!"#$%&"'( * &++,- .$!/#0" 4
Anbochi, Atillo, Weigand
Taken from Rhys Alexeis reviewer. Digests from upperbatch.


Ernani Trinos (deceased) was the husband of private respondent Julita. During
Ernanis lifetime, he applied for a healthcare coverage with Philamcare. In the
application form he was asked, Have you or any of your family members ever
consulted or been treated for high blood sugar, heart trouble, diabetes, cancer,
liver disease, asthma or peptic ulcer? He answered no.
The application was approved for a period of one year. Under the coverage, he
was entitled to hospitalization benefits, out patient benefit (like physical
examinationsetc.). The agreement was extended for another year after its
termination. During this period, Ernani suffered a heart attack and was confined
at the Manila Medical Center. Julita tried to claim from the benefits under the
agreement but Philamcare refused.
Philamcare alleges that the agreement was void because there was
concealment of Ernanis medical history. The Doctors at MMC found that Ernani
was hypertensive, diabetic and asthmatic which was contrary to what he
answered in the application.
Ernani was brought home due to financial difficulties but was later admitted to
the Chinese General Hospital after he felt very weak. He died there. Julita filed
an action for damages against Philamcare. The lower court and the CA
sustained Julita. Philamcare avers that the healthcare agreement is not an
insurance contract, hence the incontestability clause does not apply.

Philamcares basis for saying that the agreement is not an insurance contract
are: 1) the living benefits may only be enjoyed as long as he is alive upon
effectivity of the agreement and its expiration; 2) the benefits are given without
any indemnification unlike in an insurance contract; and 3) the incontestability
clause requires an effectivity period of at least 2yrs.

Issue: Whether or not the healthcare agreement is an insurance contract
considering the interest that was insured?

Held: A health care agreement is an insurance contract.

Ratio: (only important part is on what may be insured)
The IC (Insurance Code) defines a contract of insurance as an agreement
whereby one undertakes for a consideration to indemnify another against loss,
damage or liability arising from an unknown or contingent event. The following
are the requisites: 1) the insured has an insurable interest;
2) insured is subject to a risk of loss;
3) insurer assumes the risk;
4) assumption of risk is part of a scheme to distribute actual losses among a
large group of persons bearing a similar risk; and
5) insured pays a premium.

Section 3 of the Insurance Code states that any contingent or unknown event,
whether past or future, which may damnify (I guess this means affect
negatively) a person having an insurable interest against him, may be insured
against. Every person has an insurable interest in the life and health of himself.
Hence, the healthcare agreement was an insurance contract.

Every person has an insurable interest in the life and health: 1) of himself, of his
spouse and of his children; 2) of any person on whom he depends wholly or in
part for education or support, or in whom he has a pecuniary interest; 3) of any
person under a legal obligation to him for the payment of money, respecting
property or service, of which death or illness might delay or prevent the
performance; and 4) of any person upon whose life any estate or interest
vested in him depends.

In the case at bar, the insurable interest of respondents husband in obtaining
the health care agreement was his own health. The health care agreement was
in the nature of non-life insurance, which is primarily a contract of indemnity.
Once the member incurs hospital, medical or any other expense arising from
sickness, injury or other stipulated contingent, the health care provider must pay
for the same to the extent agreed upon under the contract.

(Other doctrines: To warrant recission, the fraudulent intent of the insured
must be proven. In this case, the alleged misrepresentation was to his answer
of No regarding his medical history. The Court sees the answer to this
question to be based largely on opinion. Ernani, not being a medical doctor,
could not have factually known his medical condition.)

Filipinas Cia. De Seguros v Christern Huenfeld & Co.
Respondent company Christern obtained a fire policy from Filipinas Cia
covering merchandise contained in a building in Binondo (premium of P100k
already paid). During the Japanese occupation, the building and insured
merchandise were burned. Christern submitted to the insurer Filipinas Cia its
claim under the policy. The salvage goods were sold at public auction and after
deducting their value, the total loss suffered was fixed at P92,650.

Filipinas Cia refused to pay on the ground that the policy had ceased to be in
force on the date the US declared war against Germany, Christern being
controlled by German subjects and Filipinas Cia being a company under
American jurisdiction when the policy was issued in 1941. Nevertheless,
Filipinas paid Christern pursuant to the order of the Dir. of the Bureau of
Finance.


!"#$%&"'( * &++,- .$!/#0" 5
Anbochi, Atillo, Weigand
Taken from Rhys Alexeis reviewer. Digests from upperbatch.

Filipinas Cia filed the present action to recover the sum it paid. Its theory is that
the insured merchandise were burned after the policy had ceased to be
effective because of the war between US & Germany, and that payment was
made under pressure. The CFI dismissed the case, CA affirmed.

The CA overruled the contention that Christern became an enemy when the US
declared war against Germany, relying on English and American cases which
held that a corporation is a citizen of the country or state by and under the law
of which it was created or organized. It rejected the theory that the nationality of
a private corporation is determined by the character or citizenship of its
controlling stockholders.

Issues:
W/N Christern became an enemy corporation YES
W/N the insurance policies ceased to take effect YES

Ratio:
The English and American cases relied upon by the CA have lost their force in
view of the latest decision of the US SC in Clark v. Ueber-see Finanz
Korporation, in which the control test has been adopted. A corporation was
subject to enemy legislation when it was controlled by enemies, namely
managed under the influence of individuals or corporations themselves
considered as enemies (piercing the corporate veil).

In Haw Pia v. China Bank, the SC held that the latter came within the meaning
of the word enemy as used in the Trading with the Enemy Acts of civilized
countries not only because it was incorporated under the laws of an enemy
country, but because it was controlled by enemies.

The Philippine Insurance Law, sec. 8, provides that anyone except a public
enemy may be insured. It stands to reason that an insurance policy ceases to
be allowable as soon as an insured becomes a public enemy.

Having become a public enemy, the insurance policy issued to Christern had
ceased to be valid and enforceable and was not entitled to any indemnity under
the policy. However, elementary rules of justice require that the premium paid
for the period covered by the policy should be returned to Christern.

San Miguel v Law Union and Rock Insurance and Henry Harding

This action was instituted by petitioner, as mortgagee-creditor of Henry Harding,
to recover the face value of the insurance policies covering properties [not
mentioned what kind] which were destroyed by fire. The total face value of the
policies was 15,000 chargeable against respondent insurance company and
Filipinas Compania De Seguros.
The balance Harding owed to San Miguel was 4K +++. Respondent insurance
companies agreed to pay petitioner 4k+++ representing the balance of the
mortgage debt. However, the insurance companies refused to pay Harding, the
true and real owner, of the insured properties the difference between the
balance of the mortgage debt and the face value of the policies [ie. 15,000
4,000+++]
The contention of respondent insurance companies was that their liability was
limited to the insurable interest of the plaintiff over the properties.
The issue is whether Harding could recover the deficiency [between petitioners
mortgage credit and the face value of the policies]

HELD: Insurance companies win.
Harding cannot recover. This is because the interest that was insured was only
the mortgage interest of petitioner as creditor-mortgagee ie. which was only up
to 4k+++. This meant that the insurance companies cannot absolutely be made
to pay more than 4k+++. The properties were originally owned by Dunn (before
he assigned it to Harding). Dunn obtained a loan of 10,000 from petitioner and
mortgaged the properties. Thereafter, Dunn asked petitioner to insure ALL the
interests in the properties and charge the premium paid to Dunn. Petitioner thus
obtained the insurance of herein respondent companies. HOWEVER, during the
negotiations for the insurance, petitioner represented to the companies that its
interest over the properties was only that of a mortgagee to the extent of the
balance of the mortgage debt. Thus only such portion of the entire interest was
insured. It follows therefore that the owners interest is not insured.
Furthermore, since Harding only stepped into the shoes of Dunn as assignee of
the properties, he cannot have better rights than Dunn. He too cannot collect
from defendant companies.

Grepalife v CA and Luterio
A contract of group life insurance was executed between Great Pacific Life
Assurance Corp (Grepa Life) and Devt Bank of the Phils (DBP), where Grepa
Life agreed to insure the lives of eligible housing loan mortgagors of DBP.

Dr. Leuterio, a housing debtor of DBP, applied for membership in the group life
insurance plan and in the application form, he stated that he is in good health,
never having consulted a physician for a heart condition, high blood pressure,
cancer, diabetes, etc. Thus, Grepa Life issued him an insurance coverage to
the extent of his DBP mortgage indebtedness amounting to 86K.

Less than a year later, Dr. Leuterio died due to massive cerebral hemorrhage,
and DBP submitted a death claim to Grepa Life. Grepa Life denied the claim,

!"#$%&"'( * &++,- .$!/#0" 6
Anbochi, Atillo, Weigand
Taken from Rhys Alexeis reviewer. Digests from upperbatch.

alleging that Dr. Leuterio was not healthy when he applied for an insurance
coverage. Grepa Life said that such concealment justified the denial of DBPs
claim.

Dr. Leuterios widow filed a complaint for specific perf. with damages. RTC ruled
for her, ordering Grepa Life to pay DBP 86k, and the CA sustained this
decision, hence this. Grepa Life argues that because the complaint was
instituted by the widow, who is not the real party in interest, the RTC had no
jurisdiction over the case. DBP was the indispensable party who wasnt joined
in the suit.

Issue: w/n Grepa Life can be made liable to DBP as a beneficiary in a group life
insurance contract based on a complaint filed by the widow of the decedent-
mortgagor

Ruling: yes.
The rationale of a group insurance policy of mortgagors, otherwise known as
the mortgage redemption insurance, is a device for the protection of both the
mortgagee and the mortgagor. On the part of the mortgagee, it has to enter into
such form of contract so that in the event of the unexpected demise of the
mortgagor during the subsistence of the mortgage contract, the proceeds from
such insurance will be applied to the payment of the mortgage debt, thereby
relieving the heirs of the mortgagor from paying the obligation. Ample protection
is also given to the mortgagor so that in the event of death, the mortgage
obligation will be extinguished by the application of the insurance proceeds to
the mortgage indebtedness. Where the mortgagor pays the insurance premium
under the group insurance policy, making the loss payable to the mortgagee,
the insurance is on the mortgagors interest, and the mortgagor continues to be
a party to the contract. In this type of policy insurance, the mortgagee is simply
an appointee of the insurance fund, such loss-payable clause does not make
the mortgagee a party to the contract. (this explains Sec 8 of the Code).

And since a policy of insurance may pass by transfer, will or succession to any
person, the Leuterio widow may filey the suit against Grepa Life.

HOWEVER, after the CA ruling and before the disposition of this case, it turns
out, DBP already foreclosed on the mortgage. Thus, the insurance proceeds
shall now inure to the benefit of the heirs of the deceased or his beneficiaries.
DBP should not unjustly enrich itself.

On the concealment issue: Grepa Life was not able to substantiate this claim,
no autopsy was conducted, the real cause of Luterios death is still unknown.

Col. Castro v Insurance Commissioner
Castro applied for an insurance policy on the life of his driver, Johnny Terrenal.
Insular Life issued the policy. The policy provided for a double indemnity in case
of accidental death. 3months after the issuance of the policy, Terrenal was
allegedly shot to death by unknown persons. Castro claimed for the benefits but
Insular Life refused on the ground that the insurance policy issued was void.
Issue: Whether the insurance policy was void?
Held: Yes! Castro cannot claim the benefits.
Ratio: Authorities have held, public policy does not permit one having no
insurable interest to procure a policy of insurance upon the life of [another] and
pay the premium as speculation or on a chance of collecting money where
the required relationship is lacking, the person to be benefited by the policy is
interested instead on the death of the of the insured rather than on his life. In
short, the person to be benefited has no business making sure that insureds life
continued (meaning, wala naman siya mapapala kung mabuhay ng matagal
yung insured so bakit niya ikukuha ng insurance yung tao? Dubious yung
purpose diba?)
In this case, it is obvious that Castro has no insurable interst over the
life of his driver. Noticeable is the fact that the policy covers 20yrs of Terrenals
life which is weird considering that drivers come and go. He also was not able to
establish any pecuniary loss on the death of his driver, instead, he had more to
gain.
Even their relationship as employer and employee is not sufficient to
give Castro an interest over the life of his driver. It would be fine had the
employee taken the insurance policy himself and just named his employer his
beneficiary. However, if the employer procures the policy on his own account on
the life of an employee, he has no insurable interest and the policy is
considered void.
It is required that the person taking out insurance as beneficiary has a
legal claim from the insured for services or that he has a reasonable right to
expect some pecuniary advantage from the continuance of the life of the
insured or to fear the loss of his death.
Even if we assume that Castro had an insurable interest, as claimant,
he still failed to prove that his driver died because of an accident.

Lincoln National Life v San Juan

!"#$%&"'( * &++,- .$!/#0" 7
Anbochi, Atillo, Weigand
Taken from Rhys Alexeis reviewer. Digests from upperbatch.

Lincoln Insurance Co. is seeking the rescission of 5 life insurance policies taken
on the life of Misterioso San Juan, with his wife and Luis Parco as beneficiaries,
on the basis of false representations and concealment of material facts made by
the defendants San Juan and Parco in their applications. It was alleged that the
insured contracting party lacked insurable interest on the life of the person
insured. Another ground alleged was the lack of insurable interest of defendant
Luis Parco on the life of San Juan.
Defendants San Juan and Parco aver that Lincoln had conducted investigations
before acting on their application and was aware of these false representations
but nevertheless, issued the policies (which amounts to a waiver). They allege
that Lincoln filed these complaints in order to avoid liabilities in their policies
after their rights as beneficiaries have accrued (as a result of San Juans death).
The RTC declared the policies to be null and void.
Issue: W/N the policies are null and void YES
Ratio: There is not a shred of evidence adduced by appellants Mrs. San Juan
and Parco that Lincoln had previous knowledge of said false representations
(appellants have the burden of proving this).
The CA considers the 5 insurance policies as wagering or highly speculative
contracts of insurance, which are void for reasons of public policy, and not
being based on the existence of insurable interests on the part of Parco on the
life of San Juan, the issuance having been brought about and procured through
false affirmations, representations, and concealment:
1. That San Juan was a proprietor and fish merchant for 10 years, when in fact
he had no real estate property nor was it shown that he really was a merchant;
2. That San Juan had no employer but himself, when in fact he is employed as
a tenant in Parcos coconut farm;
3. That his income exceeded P5k when he yearly earnings are barely that of a
farm laborer; and
4. That there was no pending application for life insurance nor was any issued
at the time of application, when in fact there were several already issued by
other companies.
Lack of Insurable Interest
The policies lack the element of insurable interest. San Juan could not have
afforded the insurance policies, whereas there is abundant evidence to show
that the real party in interest in these policies was Parco, and that San Juan
was a mere nominal party, since Parco paid for them.
Evidence also shows that the policies were procured as part of a general plan to
defraud Lincoln and other companies. The discovery of a severed human head
in an advance state of decomposition left intentionally in a jeepney and being
pointed out as San Juan erases any doubt that the insurance policies belong to
the class designed to perpetrate a massive fraud against the companies.
The lower court refused to return the paid premiums to Parco on the ground that
he appears to have strongly engineered the killing of a person. This is mere
suspicion, hence, the CA ordered their return.

Insular Life v Ebrado
Buenaventura Ebrado was married to Pascuala with whom he had 6 legitimate
children. However, during this marriage he was living with his mistress Carponia
Ebrado with whom he had 2 children. Buenaventura was issued a whole life
policy insurance by Insular Life in which he named Carponia (mistress) as his
beneficiary.
Buenaventura was hit by a falling branch of a tree and died. Carponia
filed her claim for the proceeds of the insurance policy with Insular Life who was
constested by Pascuala (real wife). Insular life filed an interpleader as to whom
it should pay the insurance proceeds.
Issue: Who between the wife and the mistress has the right to receive the
insurance proceeds?
Held: The proceeds must go to the estate of the deceased.
Ratio: The Insurance Act, later superseded by the Insurance Code does not
explicitly answer the question posed. However, Art. 2011 of the NCC provides
that The contract of insurance is governed by special laws. Matters not
expressly provided for shall be regulated by this Code. Under Art.2012 of
NCC, a person forbidden from being a donee cannot be made a beneficiary of a
life insurance by the person who cannot donate to him/her. And under Art. 739,
a donation between persons who where guilty of adultery or concubinage at the
time of donation is considered void (also those made between persons found
guilty of the same criminal offense).
A life insurance policy and a donation are similar in that they both founded on
liberality. A beneficiary is like a donee because from the premiums the insured

!"#$%&"'( * &++,- .$!/#0" 8
Anbochi, Atillo, Weigand
Taken from Rhys Alexeis reviewer. Digests from upperbatch.

has paid out of liberality, the beneficiary receives the proceeds. Hence, Art.739
applies to insurance contracts. Policy considerations and dictates of morality
rightly justify the institution of a barrier between common law spouses in record
to Property relations since it encroaches upon the rights of the legitimate family.
So long as marriage remains the cornerstone of our family law, reason and
morality alike demand that the disabilities attached to marriage should likewise
attach to concubinage (recall sales that husband and wife are prohibited from
donating/selling to one another because of the influence one exerts over the
other). Conviction for adultery or concubinage is also not needed before the
prohibition under Art.739 may apply.
If the beneficiary is disqualified, the proceeds shall be paid to the estate of the
deceased.

Philamlife v Pineda
On Jan. 15 1963, Dimayuga processed an ordinary life insurance policy from
Philamlife and designated his wife and children as irrevocable beneficiaries.
On Feb. 22, 1980, Dimayuga filed a petition in court to amend the designation
of the beneficiaries in his policy from irrevocable to revocable.
Lower Court granted the petition.

Issue: WON the court erred in granting Dimayugas petition.
Held: YES.
Under the Insurance Act, the beneficiary designated in a life insurance
contract cannot be changed without the consent of the beneficiary because he
has a vested interest in the policy. The policy contract states that the
designation of the beneficiaries is irrevocable. Therefore, based on the said
provision of the contract, not to mention the law then applicable, it is only with
the consent of all the beneficiaries that any change or amendment in the policy
may be legally and validly effected. The contract between the parties is the law
binding on them.
_________________________________________

Harvardian Colleges v County Bankers Insurance Corp.
Harvardian is a family corporation, the stockholders of which are Ildefonso Yap,
Virginia King Yap and their children.
Prior to Aug. 9, 1979, an agent of Country Bankers proposed to Harvardian to
insure its school building. Although at first reluctant, Harvardian agreed.
Country Banks sent an inspector to inspect the school building and agreed to
insure the same for P500,000 for which Harvardian paid an annual premium of
P2,500.
On Aug. 9, 1979, Country Bankers issued to Harvardian a fire insurance policy.
On March 12, 1980, (39 days before I was born hehehehe )during the
effectivity of said insurance policy, the insured property was totally burned
rendering it a total loss.
A claim was made by plaintiff upon defendant but defendant denied it
contending that plaintiff had no insurable interest over the building constructed
on the piece of land in the name of the late Ildefonso Yap as owner.
It was contended that both the lot and the building were owned by Ildefonso
Yap and NOT by the Harvardian Colleges.

Issue: WON Harvardian colleges has a right to the proceeds.
Held: Harvardian has a right to the proceeds.
Regardless of the nature of the title of the insured or even if he did not have
title to the property insured, the contract of fire insurance should still be upheld if
his interest in or his relation to the property is such that he will be benefited in its
continued existence or suffer a direct pecuniary loss from its destruction or
injury. The test in determining insurable interest in property is whether one will
derive pecuniary benefit or advantage from its preservation, or will suffer
pecuniary loss or damage from its destruction, termination or injury by the
happening of the event insured against.
Here Harvardian was not only in possession of the building but was in fact
using the same for several years with the knowledge and consent of Ildefonso
Yap. It is reasonably fair to assume that had the building not been burned,
Harvardian would have been allowed the continued use of the same as the site
of its operation as an educational institution. Harvardian therefore would have
been directly benefited by the preservation of the property, and certainly
suffered a pecuniary loss by its being burned.
______________________________________________________
Ong Ling Sing v FEB Leasing
A lease agreement was executed between FEB and JVL Food Products for the
lease of motor vehicles and equipment. Petitioner executed an individual
guaranty agreement, with FEB to guarantee the prompt and faithful
performance of the terms and conditions of the aforesaid lease agreement.
Corresponding Lease Schedules with Delivery and Acceptance Certificates[5]
over the equipment and motor vehicles formed part of the agreement. Under

!"#$%&"'( * &++,- .$!/#0" 9
Anbochi, Atillo, Weigand
Taken from Rhys Alexeis reviewer. Digests from upperbatch.

the contract, JVL was obliged to pay FEB an aggregate gross monthly rental of
P170,494.00. JVL defaulted, and despite demand letters sent by FEB, JVL
failed to pay. A complaint for sum of money was filed in the RTC against
petitioner, JVL, and a John Doe. JVL and Lim claimed that they were made to
believe that when full payment was effected, a Deed of Sale will be executed by
FEB as vendor in favor of JVL and Lim as vendees. FEB purportedly assured
them that documenting the transaction as a lease agreement is just an industry
practice and that the proper documentation would be effected as soon as full
payment for every item was made. They also contended that the lease
agreement is a contract of adhesion and should, therefore, be construed
against the party who prepared it, i.e., FEB. The Court ruled that the plaintiff can
only recover the unpaid balance of the price of the returned vehicles, because
of the previous payments made by the defendants for the reasonable use of the
units, specially so, as it appears, these returned vehicles were sold at auction
and that the plaintiff can apply the proceeds to the balance. However, with
respect to the unreturned units and machineries still in the possession of the
defendants, it is this Courts view and so hold that the defendants are liable
therefore and accordingly are ordered jointly and severally to pay the price
thereof to the plaintiff together with attorneys fee and the costs of suit in the
sum of Php25,000.00. Notice of appeal was filed by FEB.
The CA reversed the lower court judgment and ordered JVL Food Products and
Vicente Ong Lim, Jr. to solidarily pay FEB Leasing and Finance Corporation
the amount of Php3,414,468.75, with interest at the rate of twelve percent
(12%) per annum starting from the date of judicial demand on 06 December
2000, until full payment thereof.
Issue: (relevant to insurance) W/N Lessee JVL has an insurable interest in the
leased property.
Held: Yes. The stipulation in Section 14 of the lease contract, that the
equipment shall be insured at the cost and expense of the lessee against loss,
damage, or destruction from fire, theft, accident, or other insurable risk for the
full term of the lease, is a binding and valid stipulation. Petitioner, as a lessee,
has an insurable interest in the equipment and motor vehicles leased. Section
17 of the Insurance Code provides that the measure of an insurable interest in
property is the extent to which the insured might be damnified by loss or injury
thereof. It cannot be denied that JVL will be directly damnified in case of loss,
damage, or destruction of any of the properties leased.
In the financial lease agreement, FEB did not assume responsibility as to the
quality, merchantability, or capacity of the equipment. This stipulation provides
that, in case of defect of any kind that will be found by the lessee in any of the
equipment, recourse should be made to the manufacturer. The financial
lessor, being a financing company, i.e., an extender of credit rather than an
ordinary equipment rental company, does not extend a warranty of the fitness of
the equipment for any particular use. Thus, the financial lessee was precisely in
a position to enforce such warranty directly against the supplier of the
equipment and not against the financial lessor. We find nothing contra legem or
contrary to public policy in such a contractual arrangement. Petition thus
denied, and CA decision affirmed.
____________________________________________________
Lampano v Barretto
Mariano Barretto constructed a house for Placida Jose. After its completion,
Mariano took out an insurance policy upon it in his own name with Placidas
consent. Later, Placida sold the house to Antonina Lampano. However, the
house was destroyed by a fire. At this time, Antonina still owed Placida P2K,
while Placida still owed Mariano P2K. Mariano was able to collect P3.6K from
the insurance company.
Lampano alleged in her complaint that she had the right to the proceeds of the
insurance policy. According to her, she and Placida had a verbal agreement
that the insurance policy will be delivered to her and it was only after the fire
occurred that she found out the insurance policy was under the name of
Mariano. Placida denied the verbal agreement.
Issue: Who has the right to the proceeds of the insurance policy?
Held: Mariano Barretto has the right!
Ratio: If Mariano had an insurable interest in the house, he could insure this
interest for his sole protection. Under the insurance contract, the insurance
policy would be payable to the insured regardless of the extent of his interest as
long as he had one at the time of the making of the contract, and he still has an
interest at the time of incident [in this case fire]. If there are different persons
who have different interests in the same property, an insurance taken by one
person based on his own interest does not benefit the others.
A contract of insurance made for the insurer's (insured) indemnity
only, as where there is no agreement, express or implied, that it shall be for the
benefit of a third person, does not attach to or run with the title to the insured
property on a transfer thereof personal as between the insurer and the insured.
In such case strangers to the contract cannot require in their own right any

!"#$%&"'( * &++,- .$!/#0" 1:
Anbochi, Atillo, Weigand
Taken from Rhys Alexeis reviewer. Digests from upperbatch.

interest in the insurance money, except through an assignment or some
contract with which they are connected."
Barretto clearly had an insurable interest in the house. He constructed it and
provided for the materials.
___________________________________________________
Lopez v Del Rosario
Mrs. Del Rosario was the owner of a bonded warehouse. She was engaged in
the business of a warehouse keeper and stored copra and other merchandise in
the said building. Lopez was the holder of 14 warehouse receipts, with a
declared value of P107k . The receipts provided for insurance at the rate of 1%
per month of the declared value. It was admitted that Lopez paid insurance up
to May 1920, but not thereafter.
Del Rosario secured insurance on the warehouse and its contents with 5
insurance companies in the amount of P404k. The warehouse and its contents
were destroyed by fire, while some copra was salvaged (amounting to P49k).
Del Rosario and the insurance companies submitted the matter to arbitration.
The arbiters allowed Del Rosario to collect P363k with the addition of the money
received from the salvaged copra. Del Rosario seemed to have satisfied all of
the persons who had copra stored in warehouse except for Lopez. They failed
to compromise on an amount, since Lopez contends that he should receive no
less than P88k.
Del Rosario alleges that Lopezs right to the insurance money had been
forfeited when he failed to pay the insurance provided for in the warehouse
receipts.
Issue: Whether or not Lopez is entitled to the P88k YES, but with deductions
Ratio: The law is that a policy effected by a bailee and covering by its terms his
own property and property held in trust, inures, in the event of a loss, equally
and proportionately to the benefit of all the owners of the property insured. This
is true even if the owner of the stored goods did not request or know of the
insurance, and did not ratify it before the payment of the loss. The
warehouseman is liable to the owner of such stored goods for his share. In
effect, Del Rosario acted as the agent of Lopez in taking out the insurance on
the contents of the warehouse.
Further, Del Rosario acknowledged her responsibility to the owners of the
stored merchandise. The award of the arbitrators did not only cover Del
Rosarios warehouse but also the products stored therein. Insurable interest of
the depositary in the goods deposited, is the extent of his liability to the owner of
the goods in his warehouse.
Lopezs right to the insurance has not been forfeited by failure to pay the
insurance provided in the warehouse receipts. There is no proof showing that
he even ordered its cancellation, nor did he refuse to pay the insurance when
the bills were presented to him.
The SC ruled that Lopez is entitled to the P88k, but minus the amounts for his
share of the expenses and those due for insurance and storage.
____________________________________________________________
Cha v CA
Spouses Cha as lessees entered into a lease contract with private respondent
CKS Devt Corp (CKS) as lessor. One of the stipulations in their contract was
that lessees shall not insure against fire any merchandise or chattels placed at
any stall of space in the leased premises without the written consent of the
lessor. Should the lessee obtain the insurance therefor, the policy is deemed
assigned and transferred to the lessor (no mention of what was leased and
what merchandise were involved).
Notwithstanding this stipulation, spouses Cha insured their merchandise inside
the leased premises against fire for 500k with United Insurance without CKSs
written constent..
When contract was about to expire, fire broke out in the leased premises. CKS
learned of the insurance and wrote to United asking that the proceeds of the
insurance be paid to CKS directly. United refused and CKS sued both the
spouses Cha and United.
RTC and CA ruled for CKS, ordering United to pay the proceeds of the
insurance to it. Hence this petition.
Issue: W/N Cha spouses can claim from United (w/n the stipulation in the
contract is valid)
Ruling: yes, Cha spouses can claim; contract stipulation is invalid.

!"#$%&"'( * &++,- .$!/#0" 11
Anbochi, Atillo, Weigand
Taken from Rhys Alexeis reviewer. Digests from upperbatch.

Insurable interest in the property insured must exist at the time the insurance
takes effect and at the time the loss occurs (Sec 18). This rule is based on
public policy: a person should not take out an insurance policy on property upon
which he has no insurable interest and collect the proceeds of said policy in
case of loss of property. In such a case, the contract of insurance is a mere
wager and is void under the Insurance Code (Sec 25).
In this case, CKS has no insurable interest in the goods and merchandise inside
the leased premises. Therefore, CKS cannot be validly a beneficiary of the fire
insurance policy taken by the Spouses Cha over their merchandise. The
provision for the automatic assignment of policy is void for being contrary to law
and public policy. Proceeds of the insurance policy belong to the spouses.
Automatic transfer of the insurance contract is not allowed, most especially id
the transferee has no insurable interest in the subject matter, and cannot bring
an action to enforce the insurance contract, which is unenforceable as to him.
The liability of the Cha spouses for violating their lease contract with CKS is a
separate issue which cannot be resolved in this case.
___________________________________________
Tai Tong Chua Che v Ins. Commissioner
Palomo obtained a loan from Taitong for 100T. To secure this, he mortgaged a
parcel of land with a building. Taitong insured the mortgaged property with
Travelers Multi-Indemnity Corp for 100T.
The insured property was razed by fire. Taitong claimed the proceeds from the
insurance company.
Travelers refused to pay, claiming that Taitong had no more insurable interest in
the property since Palomo had allegedly paid the mortgaged debt already.

Issue: W/N Taitong can collect the proceeds.
Held: Yes. The allegation of the insurance company that the debt had already
been paid was NOT proved. Tai Tong on the other hand presented evidence,
namely the contract of mortgage which does not appear to have been canceled
or released.
__________________________________________
Bachrach v British American Assurance Co.
A fire insurance policy was issued by British American to Bachrach. The fire
policy provides that Bachrach paid to defendant the sum of PhP2,000 for
insuring against loss or damage by fire, the following properties: goods,
belonging to a general furniture store, such as iron and brass bedsteads, toilet
tables, chairs, ice boxes, bureaus, washstands, mirrors, and sea-grass
furniture, the property of the assured, in trust, on commission or for which he is
responsible, whilst stored in the ground floor and first story of house and
dwelling No. 16 Calle Martinez, district 3, block 70, Manila, built, ground floor of
stone and or brick, first story of hard wood and roofed with galvanized iron. At
the back of the policy, it was stated that a calalac automobile is also included
thereunder.
The goods were destroyed by a fire, but the company refused to pay the
proceeds contending that the plaintiff maintained a paint and varnish shop in the
said building where the goods which were insured were stored, transferred his
interest in and to the property covered by the policy to H. W. Peabody & Co. to
secure certain indebtedness due and owing to said company, and also that the
plaintiff had transferred his interest in certain of the goods covered by the said
policy to one Macke, to secure certain obligations assumed by the said Macke
for and on behalf of the insured. That the sanction of the said defendant had not
been obtained by the plaintiff, as required by the said policy.
It also contends that immediately preceding the outbreak of the alleged fire,
willfully placed a gasoline can containing 10 gallons of gasoline in the upper
story of said building in close proximity to a portion of said goods, wares, and
merchandise, which can was so placed by the plaintiff as to permit the gasoline
to run on the floor of said second story, and after so placing said gasoline, he,
the plaintiff, placed in close proximity to said escaping gasoline a lighted lamp
containing alcohol, thereby greatly increasing the risk of fire, and plaintiff failed
to provide for proof of the said loss.
The lower court found that the defendant was liable to the plaintiff and rendered
a judgment against the defendant for the sum of P9,841.50, with interest for a
period of one year at 6 per cent, making a total of P10,431.99, with costs.
Issue: Is the defendant liable to pay the proceeds of the policy to plaintiff?
Held: The policy was NOT forfeited due to the strong paints and varnishes.
There was no express provision pertaining to it and these paints and varnishes
are incidental to the business of the insured to keep the furniture in a saleable
condition. The gasoline stored within the premises was in the reservoir of the
car and thus does not violate any provision in the policy. There is no express
prohibition against the execution of a chattel mortgage on the property insured.
Lower court judgment should be affirmed.
Sections 26-35

!"#$%&"'( * &++,- .$!/#0" 12
Anbochi, Atillo, Weigand
Taken from Rhys Alexeis reviewer. Digests from upperbatch.

Argente v West Coast Insurance 51 Phil. 725
Petitioner obtained a joint life insurance for himself and his wife Vicenta De
Ocampo with West Coast for Php 2,000. The application forms were written
entirely by the West Coast agent upon information given to him by the spouses.
The only physical participation of the spouses with respect to the application
form was their affixation of signature.
Thereafter, spouses underwent a medical examination before Dr. Sta. Ana,
West Coasts physician wherein several question were propounded to the
spouses. Among these questions was whether each of them had any sort of
brain ailment. To this question, the spouses answered NO.
Meanwhile spouses filed an amended application increasing the insurance to
15,000. A few months after the policy was issued to them, Vicenta died of
CEREBRAL APOPLEXY.
Apparently, before the issuance of the policy Argente was admitted to the PGH
where he was diagnosed with cerebral congestion and Bells Palsy. Vicenta was
also admitted to Mary Chilles hospital where she was diagnosed with manic-
depressive psychosis and thereafter psycho-neurosis.
Because of Vicentas death, Argente filed a claim against West Coast. West
Coast argues that there was concealment on the part of the spouses. Argente
argues that they (referring to spouses) have been truthful in their disclosures
and that West Coasts agent and Dr. Sta Ana conspired for some reason not to
include the real answers on the medical report. RTC ruled in favor of West
Coast.
Issue: W/N there was really concealment on the part of the spouses thereby
barring them from recovering from the life insurance policy
Held: West Coast wins.
The contention of Argente that the misrepresentations were too trivial to
avoid the life policy is untenable. In cases concealment, the concealment
must be (1) of information which is material (2) fraudulent. In this case the
answers to the question regarding the health conditions of the spouses
were relevant. Had the spouses disclosed the real condition of their health,
West Coast would not have undertaken the risk of insuring their lives.
The other contention of Argente to the effect that West Coast is precluded
from rescinding the insurance contract because the Insurance Act says that
rescission must be made prior to the institution of action if untenable. This
is because the fact is, prior to the institution of the present action, West
Coast had already notified Argente that the policy was avoided because of
concealment and even offered to return the premium paid in exchange for
the surrender of the policy. This, according to the court, is tantamount to
rescission of the insurance contract.

Saturnino v. Philam Life 117 Phil. 330
Upon submission of the application form (September 1957), Philam Life issued
a non-medical insurance policy with 20-year endowment to the deceased,
Estefania Saturnino and the latter paid the first premiuim. This type of policy
dispenses with the medical examination usually required in life policies.
1 year later (September 1958) the deceased died of pneumonia secondary to
influenza. It appears that 2 months prior to the issuance of the policy, deceased
was operated on for cancer, involving the complete removal of her right breast.
According to her surgeon, she could not be considered definitely cured because
her ailment was the malignant type.
Herein petitioners (husband and son of the deceased) tried to claim the face
value of the policy from Philam Life but it refused, stating that there were
material misrepresentations. Notwithstanding the fact of her operation, the
deceased did not make a disclosure thereof in her application. She actually
stated therein that she did not have, nor ever had, cancer or other tumors; that
she had not consulted any physician, undergone any surgery operation or
suffered any injury within the preceding 5 years; and that she had never been
treated nor did she ever have illness or disease peculiar to her sex, particularly
of the breast, uterus, ovaries and menstrual disorders.
Petitioners sued. Trial court dismissed the complaint, hence this appeal.
Issue: WN the insured made false representations of material facts as to avoid
the policy
Ruling: Yes. Case dismissed.
Petitioners contend that there was no fraudulent concealment of the truth since
the deceased herself did not know that the disease for which she was operated
on was cancer (doctor never told her). In the first place, the concealment of the
fact of operation itself was fraudulent. Secondly, to avoid a policy it is not
necessary to show actual fraud on the part of the insured. A
concealment, whether intentional or not, entitles the insurer to rescind the
contract of insurance, concealment being defined as negligence to
communicate that which a party knows and ought to communicate. As

!"#$%&"'( * &++,- .$!/#0" 13
Anbochi, Atillo, Weigand
Taken from Rhys Alexeis reviewer. Digests from upperbatch.

was held in Argente, the basis for the rule vitiating the contract in cases of
concealment is that it misleads or deceives the insurer into accepting the risk
upon a false basis that does not exist.
Petitioners also contend negligence on the part of Philam Life, since in the
application, the deceased agreed to submit herself to a medical examination if
in Philam Lifes opinion such examination was necessary. No such negligence
can be imputed to Philam Life because the insured misrepresented her health
that Philam Life no longer thought a medical examination was necessary.
_______________________________________________________________
____
Fieldmens Insurance vs Songco 25 SCRA 70
Facts: Federico Songco, who was only able to reach 1
st
grade, owned a
private jeepney. On the other hand, Benjamin Sambat is an agent of Fieldmens
Insurance. While Sambat was inducing Federico to obtain an insurance policy
under the companys Common Carriers Liability Insurance Policy, Amor
Songco (Federicos son) butted in and reminded Sambat that they cannot get
the said insurance because their jeepney was private and not for passenger.
Sambat however answered that whether the vehicle was privately owned or for
passengers, it did not matter because the company was not owned by the
government so they can do whatever they want so long as they believe a
vehicle is insurable. (This testimony of Amor was not rebutted during trial)
The policy was issued and later on renewed. During the renewed period, the
insured jeepney collided with a car. Federico and his son Rodolfo died while
Carlos (another son), his sons wife Angelita and a family friend sustained
injuries.
These injured parties sought to claim from the insurance company the proceeds
of the policy. Fieldmens refused on the ground that the vehicle was private and
not a common carrier. The lower court and CA ruled in favor of plaintiffs.
Issue: W/N Fieldmens Insurance is liable?
Held: Yes!
Ratio: Estoppel applies. Fieldmens agent led Federico to believe that he
could qualify his private jeepney under the common carrier policy hence, it
could not now be permitted to change its stand to the detriment of the heirs of
the insured. Fieldmens knew that the vehicle was privately owned yet they
insured the vehicle not once, but twice.
Under the terms of the policy they cannot escape liability. (There is a
clause there that provides that the insurer will indemnify the insured in the event
of an accident caused by or from the use of the motor vehicle against all sums
which the insured may be made liable to pay on account of the death or injury of
its fare paying passengers). The injured parties in this case are passengers of
the jeepney. Even if they did not pay a fare, their status as beneficiaries under
the policy is recognized.
Besides, even if ambiguous, it is settled that it will be strictly construed
against the party that caused the ambiguity. The reason for this rigid application
is the prevalence of contracts of adhesion wherein the weaker party is left to a
take it or leave it situation.
The contract of insurance is one of perfect good faith not for the
insured alone but equally so for the insurer; in fact, it is more so for the latter
since its dominant bargaining position carries with it stricter responsibility.
Insular Life v. Feliciano 73 PHIL 201
Facts:
Evaristo Feliciano filed an application with Insular Life upon the solicitation of
one of its agents.
It appears that during that time, Evaristo was already suffering from
tuberculosis. Such fact appeared during the medical exam, but the examiner
and the companys agent ignored it.
After that, Evaristo was made to sign an application form and thereafter the
blank spaces were filled by the medical examiner and the agent making it
appear that Evaristo was a fit subject of insurance. (Evaristo could not read and
understand English)
When Evaristo died, Insular life refused to pay the proceeds because of
concealment.

Issue: W/N Insular Life was bound by their agents acts.
Held: Yes.
The insurance business has grown so vast and lucrative within the past century.
Nowadays, even people of modest means enter into insurance contracts.
Agents who solicit contracts are paid large commissions on the policies secured
by them. They act as general representatives of insurance companies.

!"#$%&"'( * &++,- .$!/#0" 14
Anbochi, Atillo, Weigand
Taken from Rhys Alexeis reviewer. Digests from upperbatch.

IN the case at bar, the true state of health of the insured was concealed by the
agents of the insurer. The insurers medical examiner approved the application
knowing fully well that the applicant was sick. The situation is one in which of
two innocent parties must bear a loss for his reliance upon a third person. In
this case, it is the one who drafted and accepted the policy and consummated
the contract. It seems reasonable that as between the two of them, the one
who employed and gave character to the third person as its agent should be the
one to bear the loss. Hence, Insular is liable to the beneficiaries.
Insular life v. Feliciano 74 PHIL 4681
Facts:
Insular life filed a motion for reconsideration of the decision in the preceding
case.

Issue: W/N Insular Life was bound by their agents acts.
Held: NO. The Court reversed itself.
There was collusion between Evaristo and the agent and the medical examiner
in making it appear that Evaristo was a fit subject for insurance. When Evaristo
authorized them to write the answers for him, he made them his own agents for
that purpose and he was responsible for their acts in that connection.
If they falsified the answers for him, he could not evade liability for the
falsification. He was not supposed to sign the application in blank. He knew
that his answers would be the basis for the policy, and was required with his
signature to vouch for their truth. The judgment rendered therefore in the
preceding case is thus reversed, and Insular Life is absolved from liability.
Sun Life v CA 245 SCRA 268
John Bacani obtained a life insurance from Sunlife for PhP 100,000 with double
indemnity. The designated beneficiary was his mother Bernarda Bacani.
In the application form, to the question whether he has consulted a doctor, he
answered yes but only for a minor cough and flu. For the other questions such
as whether he underwent several lab tests, whether he was confined in the
hospital and whether he was ever diagnosed with urine, bladder or renal
disorder, he answered NO.
Thereafter, John died in a plane crash. Thus Bernarda filed a claim with Sunlife.
Sunlife, in a letter, denied the claim and enclosed therewith a check for the
premium paid by John. Sunlife contended that 2 weeks prior to the application
John was admitted to the Lung Center of the Philippines because of renal
failure.
Bernarda sued SunLife. SunLife pleaded concealment.
RTC and CA ruled in favor of Bernarda saying that John was in good faith in not
disclosing the true answers on the impression that they were not relevant. CA
said that the answers to those questions were not relevant to the cause of death
of John and because, after all, the insurance was non-medical

Issue: W/N Bernarda can claim

HELD: Sunlife wins.
Materiality of information does not depend upon the state of mind of
the insured or the actual physical events that ensue. An information is
material if its non-disclosure would lead to the insurance company
either denying the application or increasing the premium because of
heightened risk. Sunlife, in all probabilities, would have denied the
application had it been aware of Johns renal problem.
The contention of Bacani that Sunlies waiver of the medical
examination debunks the materiality of the facts concealed is
untenable. All the more that the information becomes material now that
no medical examination was conducted. Sunlife would have nothing
rely on except the truthfulness of Bacani
Anent the finding that the facts concealed had no bearing to the cause
of death of the insured, it is well settled that the insured need not die of
the disease he had failed to disclose to the insurer. It is sufficient that
his non-disclosure misled the insurer in forming his estimates of the
risks of the proposed insurance policy or in making inquiries

Ng Gan Zee v. Asian Crusader Life Assurance Corp. 122 SCRA 461
May 1962, Kwong Nam applied with Asian Crusader for a 20-year endowment
insurance on his life for 20k, with his wife Ng Gan Zee as beneficiary. The
policy was issued, he paid premiums religiously. In December 1963, Kwong
died of cancer of the liver.
Ng Gan Zee filed a claim with Asian, but it was denied on the ground of
misrepresentation. Ng Gan brought the matter to the Insurance Commissioner,
who ordered Asian to pay, but it still refused.

!"#$%&"'( * &++,- .$!/#0" 15
Anbochi, Atillo, Weigand
Taken from Rhys Alexeis reviewer. Digests from upperbatch.

Asian claims that when the insured was examined, he gave Asians medical
examiner false and misleading information as to his ailment and previous
operation. He said: operated on for a tumor of the stomach. Claims that tumor
is associated with ulcer of the stomach. Tumor taken out was hard and of a
hens egg size. Operation was 2 years ago in Chinese Gen. Hospital. Now
claims he is completely recovered. Asian points to the misrepresentation in
that it interviewed one of the doctors who treated Kwong and said his ailment
was peptic ulcer and that the specimen removed from the deceaseds body was
a part of his stomach.
Ng Gan sued and CFI ordered Asian Crusader to pay.
Issue: W/N Kwong is guilty of misrepresentation to allow Asian to rescind (main
issue); w/n Asian Crusader waived its right to information of material facts
(important to topic)
Ruling: No fraudulent misrepresentation, Asian Crusader must pay Ng Gan;
There was a waiver.
According to the Insurance Law (not the present code) the concealment must
be material and fraudulent the fact must have been intentionally withheld.
The fraudulent intent must be established by the insurer to be able to rescind
the contract. This fraudulent intent was not present. Kwong informed the
medical examiner that his tumor was associated with ulcer of the stomach. This
should be construed as an expression made in good faith of his belief as to the
nature of his ailment. It was presumed to be made by him without knowledge of
its incorrectness and without intention to mislead Asian.
The Insurance Law states that the right to information of material facts may be
waived by the terms of the insurance or by neglect to make inquiries as to such
facts. Where upon the face of the application, a question appears to be not
answered at all or to be imperfectly answered, and the insurer still issues
a policy without any further inquiry, they waive the imperfection of the
answer and render the omission to answer fully immaterial.
If the ailment of Kwong had such an important bearing on whether or not Asian
would issue the policy or not, the court cannot understand why Asian or its
medical examiners did not make further inquiries with Chinese Gen Hospital.
Truth was, Asian was too eager to accept the application to receive the
premiums. It would now be inequitable to allow Asian to avoid liability.
Section 36 - 48
Harding v Commercial Union
Facts:
Henry Harding bought a car for 2T in 1915. He then gave the car to his wife
Mrs. Harding.
While Mrs. Harding was having the car repaired at the Luneta Garage (Luneta
was an agent of Smith Bell and Co., which in turn is Commercial Unions agent),
the latter induced Mrs. Harding to insure the car with Commercial.
Mrs. Harding agreed, and Smith Bell sent an agent to Luneta Garage, who
together with the manager of Luneta, appraised the car and declared that its
present value was P3T. This amt was written in the proposal form which Mrs.
Harding signed.
Subsequently, the car was damaged by fire. Commercial refused to pay
because the cars present value was only 2.8T and not 3T.

Issue: W/N Commercial is liable.
Held: Commercial is liable.
Where it appears that the proposal form, while signed by the insured was
made out by the person authorized to solicit the insurance (Luneta and Smith
Bell) the facts stated in the proposal, even if incorrect, will not be regarded as
warranted by the insured, in the absence of willful misstatement. Under such
circumstances, the proposal is to be regarded as the act of the insurer.

Saturnino v. Philam Life Sec. 45
(Material Facts)
Facts:
Upon submission of the application form (September 1957), Philam Life issued
a non-medical insurance policy with 20-year endowment to the deceased,
Estefania Saturnino and the latter paid the first premiuim. This type of policy
dispenses with the medical examination usually required in life policies.
1 year later (September 1958) the deceased died of pneumonia secondary to
influenza. It appears that 2 months prior to the issuance of the policy, deceased
was operated on for cancer, involving the complete removal of her right breast.

!"#$%&"'( * &++,- .$!/#0" 16
Anbochi, Atillo, Weigand
Taken from Rhys Alexeis reviewer. Digests from upperbatch.

According to her surgeon, she could not be considered definitely cured because
her ailment was the malignant type.
Herein petitioners (husband and son of the deceased) tried to claim the face
value of the policy from Philam Life but it refused, stating that there were
material misrepresentations. Notwithstanding the fact of her operation, the
deceased did not make a disclosure thereof in her application. She actually
stated therein that she did not have, nor ever had, cancer or other tumors; that
she had not consulted any physician, undergone any surgery operation or
suffered any injury within the preceding 5 years; and that she had never been
treated nor did she ever have illness or disease peculiar to her sex, particularly
of the breast, uterus, ovaries and menstrual disorders.
Petitioners sued. Trial court dismissed the complaint, hence this appeal.
Issue: W/N the insured made false representations of material facts as to avoid
the policy YES
Ratio:
The Insurance Law (sec. 30) provides that Materiality is to be determined not by
the event, but solely by the probable and reasonable influence on the facts
upon the party to whom the communication is due, in forming his estimate of the
proposed contract, or in making his inquiries. Petitioners contend that the facts
subject of the representation were not material in view of the non-medical
nature of the insurance applied for, which does away with the usual requirement
of medical examination. This is without merit, since the waiver of the medical
exam renders even more material the information required of the applicant
concerning previous health and dieseases suffered, for such necessarily
constitutes an important factor which the insurer takes into consideration in
deciding whether to issue the policy or not.
Petitioners also contend that there was no fraudulent concealment of the truth
since the deceased herself did not know that the disease for which she was
operated on was cancer (doctor never told her). In the first place, the
concealment of the fact of operation itself was fraudulent. Secondly, to avoid a
policy it is not necessary to show actual fraud on the part of the insured.
A concealment, whether intentional or not, entitles the insurer to rescind
the contract of insurance, concealment being defined as negligence to
communicate that which a party knows and ought to communicate. As
was held in Argente, the basis for the rule vitiating the contract in cases of
concealment is that it misleads or deceives the insurer into accepting the risk
upon a false basis that does not exist.
Petitioners also contend negligence on the part of Philam Life, since in the
application, the deceased agreed to submit herself to a medical examination if
in Philam Lifes opinion such examination was necessary. No such negligence
can be imputed to Philam Life because the insured misrepresented her health
that Philam Life no longer thought a medical examination was necessary.


Edillon v Manila Bankers
Carmen Lapuz applied for insurance against accident and injuries with Manila
Bankers. In the application form she represented that she was born on July 11
th

1904. The application was dated April 15
th
1969. Take note that the policy
contained an overage exclusion clause whereby persons over 60 (and under
16) cannot be insured. Thereafter, the policy was issued effective for 90 days
and upon payment of the 20 peso premium. While the contract of insurance was
in force, Lapuz died of a vehicular accident. Herein petitioner Regina Edillon as
sister and designated beneficiary, filed a claim with Manila Bankers which the
latter has denied by reason of the overage exclusion clause. RTC ruled in favor
of Manila Bankers saying that the since the policy was a contract of adhesion, it
was the duty of Lapuz to know the terms and conditions of the contract she is
entering. Lapuz should have asked he refund of the premium immediately after
she should have know that she was not covered by the policy
Held: Edillon wins
Manila Bankers is estopped. Lapuz never concealed anything. It was
apparent from the application form that she was already over 65 years old
when she applied. The accident happened 45 days after the issuance of
the policy, which meant that Manila Bankers had sufficient time to correct
the mistake. In not so doing, Manila Bankers had either willingly accepted
the risk or negligently failed to make the correction for which it is entirely
to be blamed.

Tan Chay Heng v. West Coast Life Insurance Sec 45 and 48
In April 1925, West Coast accepted and approved a life insurance policy on Tan
Caeng for 10k and the sole beneficiary was Tan Chay. Tan Caeng died on May
1925, and Tan Chay submitted proofs of his death to West Coast with a claim
for the insurance.

!"#$%&"'( * &++,- .$!/#0" 17
Anbochi, Atillo, Weigand
Taken from Rhys Alexeis reviewer. Digests from upperbatch.

West Coast in its answer made a general denial (to the effect that it never
issued the policy), but in its amended answer raised the special defense of false
representation. It alleged that the policy was obtained by Tan Chay thru fraud
and deceit in confabulation with others including Dr. Locsin.
In the application, Tan Caeng designated Tan Chay as beneficiary, stating that
the latter was his nephew, but he really wasnt. Dr. Locsin (medical examiner for
West Coast) made it appear that Tan Caeng was healthy and he never used
morphine, cocaine or any other drug, when in fact, he had been suffering from
pulmonary tuberculosis for about 3 years from that time, and that he was
actually a drug addict, even having been convicted and imprisoned for drug use.
After the policy was issued, Tan Caeng shortly died of TB, but Dr. Locsin
falsified that the caused was cerebral hemorrhage.
Tan Chay turned out to be an employee of Go Chulian who is the ringleader of
a gang of malefactors engaged in procuring fraudulent life insurance policies
from West Coast (he is actually being sued for it in a different case).
The lower courts ruled for Tan Chay, mainly on the ground that an insurer
cannot avoid a policy which has been procured by fraud unless he brings an
action to rescind the policy before it is sued thereon Sec 47 of the insurance law
(now Sec 48 of the code)

Issue: W/N West Coasts action is barred by Tan Chays collection suit (in other
words, w/n Sec 47 applies)
Ruling: No. Lower court ruling is reversed, case is remanded for further
proceedings.
Rescission is the unmaking of a contract. It presupposes the existence of a
contract. Note that in this case, West Coast even denied issuing the policy at
first. Also, it never asked in its prayer that the contract be rescinded. All it did
was raise the defense that the policy was obtained through false
representations, which is NOT in the nature of an action to rescind, hence, sec
47 does not apply.
The nature of West Coasts defense is founded upon the theory that through
fraud in its execution, the policy is void, as if no contract was ever made. If all
matters alleged by defendant in its answer are true, no contract is ever made,
then there would be nothing to rescind. (important part for Sec 45) But the SC
cannot rule on the allegations, so case is remanded.
Soliman v. US Life 104 PHIL 1046
Facts:
US Life issued a 20 yr endowment life policy on the joint lives of Patricio
Soliman and his wife Rosario, each of them being the beneficiary of the other.
In March 1949, the spouses were informed that the premium for Jan 1949 was
still unpaid notwithstanding that the 31-day grace period has already expired,
and they were furnished at the same time long-form health certificates for the
reinstatement of the policies.
In Apr 1949, they submitted the certificates and paid the premiums.
In Jan. 1950, Rosario died of acute dilation of the heart, and thereafter, Patricio
filed a claim for the proceeds of the insurance.
US life denied the claim and filed for the rescission of the contract on the
ground that the certificates failed to disclose that Rosario had been suffering
from bronchial asthma for 3 years prior to their submission.

Issue: W/N the contract can still be rescinded.
Held: Yes.
The insurer is once again given two years from the date of reinstatement to
investigate into the veracity of the facts represented by the insured in the
application for reinstatement. When US life sought to rescind the contract on
the ground of concealment/misrepresentation, two years had not yet elapsed.
Hence, the contract can still be rescinded.

Tan v. CA Sec. 48
(Incontestability Clause)
Facts:
Tan applied for life insurance with Philamlife, designating his children
(petitioners) as beneficiaries, in September 1973. After 1 year and 5 months,
he died of hepatoma. Petitioners filed their claims with the company but
Philamlife denied these and rescinded the policy by reason of the alleged
misrepresntations and concealment of material facts made by Tan in his
application (at that time, he was diabetic and hypertensive and was diagnosed
to be suffering from hepatoma). The premiums paid were refunded.

!"#$%&"'( * &++,- .$!/#0" 18
Anbochi, Atillo, Weigand
Taken from Rhys Alexeis reviewer. Digests from upperbatch.

Petitioners filed a complaint with the Insurance Commissioner, which was
dismissed. CA dismissed. Before the SC, the petitioners contend that
Philamlife no longer had the right to rescind the contract of insurance as
recission must allegedly be done during the lifetime of the insured within 2 year
and prior to the commencement of an action (Sec. 48, par. 2 of the Insurance
Code).
Issue: W/N Philamlife had the right to rescind the contract of insurance YES
Ratio:
The so-called incontestability clause precludes the insurer from raising the
defense of false representations or concealment of material facts insofar as
health and previous diseases are concerned if the insurance has been in force
for at least 2 years during the insureds life time. The phrase during the
lifetime simply means that the policy is no longer considered in force after the
insured has died. The key phrase is for a period of 2 years. In this case, the
policy was only in force for a period of 1 year and 5 months. Considering that
the insured died before the 2-year period has lapsed, Philamlife is not barred
from proving that the policy is void by reason of misrepresentation by the
insured. Moreover, they rescinded the contract and refunded the premiums
before the commencement of the action.
Petitioner also contend that their could have been no concealment or
misrepresentation by their father because being a man of means, he did not
have to buy insurance. He was only pressured by the insistent salesman. They
pointed out that the SC had occasions to denounce the pressure and practice
indulged in by agents selling insurance. It would be unjust if, having been
subjected to this pressure, the assured who dies within the 2-year period should
stand charged of concealment.
The legislative answer to this argument is the incontestability clause added by
the 2
nd
paragraph of Sec. 48. The insurer has 2 years from the date of issuance
of the insurance contract or of its last reinstatement within which to contest the
policy, whether or not the insured still lives within such period. After 2 years,
the defense of concealment or misrepresentation no longer lie. Congress felt
that this was a sufficient answer to the various tactics employed by insurance
companies to avoid liability. The petitioners interpretation would give rise to the
incongruous situation where the beneficiaries of an insured who dies right after
taking out and paying for the policy, would be allowed to collect on the policy
even if there was concealment of material facts.
Cases 49 66
Enriquez v. SunLIfe 41 PHIL 269
Facts:
On Sept. 24 1917, Herrer made an application to SunLife through its office in
Manila for life annuity.
2 days later, he paid the sum of 6T to the companys anager in its Manila office
and was given a receipt.
On Nov. 26, 1917, the head office gave notice of acceptance by cable to
Manila. On the same date, the Manila office prepared a letter notifying Herrer
that his application has been accepted and this was placed in the ordinary
channels of transmission, but as far as known was never actually mailed and
never received by Herrer.
Herrer died on Dec. 20, 1917. The plaintiff as administrator of Herrers estate
brought this action to recover the 6T paid by the deceased.

Issue: W/N the insurance contract was perfected.
Held: NO.
The contract for life annuity was NOT perfected because it had NOT been
proved satisfactorily that the acceptance of the application ever came to the
knowledge of the applicant. An acceptance of an offer of insurance NOT
actually or constructively communicated to the proposer does NOT make a
contract of insurane, as the locus poenitentiae is ended when an acceptance
has passed beyond the control of the party.
NOTE: Life annuity is the opposite of a life insurance. In life annuity, a big
amount is given to the insurance company, and if after a certain period of time
the insured is stil living, he is entitled to regular smaller amounts for the rest of
his life. Examples of Life annuity are pensions. Life Insurance on the other
hand, the insured during the period of the coverage makes small regular
payments and upon his death, the insurer pays a big amount to his
beneficiaries.
_______________________________________________________________
______
Perez v. CA 323 SCRA 613 (2000)
Facts:
Primitivo Perez had been insured with the BF Lifeman Insurance Corporation
since 1980 for P20,000.00.

!"#$%&"'( * &++,- .$!/#0" 19
Anbochi, Atillo, Weigand
Taken from Rhys Alexeis reviewer. Digests from upperbatch.

In October 1987, an agent of Lifeman, Rodolfo Lalog, visited Perez in Quezon
and convinced him to apply for additional insurance coverage of P50,000.00, to
avail of the ongoing promotional discount of P400.00 if the premium were paid
annually.
Primitivo B. Perez accomplished an application form for the additional insurance
coverage. Virginia A. Perez, his wife, paid P2,075.00 to Lalog. The receipt
issued by Lalog indicated the amount received was a "deposit."
Unfortunately, Lalog lost the application form accomplished by Perez and so on
October 28, 1987, he asked the latter to fill up another application form. On
November 1, 1987, Perez was made to undergo the required medical
examination, which he passed.
Lalog forwarded the application for additional insurance of Perez, together with
all its supporting papers, to the office of BF Lifeman Insurance Corporationn in
Quezon which office was supposed to forward the papers to the Manila office.
On November 25, 1987, Perez died while he was riding a banca which capsized
during a storm.
At the time of his death, his application papers for the additional insurance were
still with the Quezon office. Lalog testified that when he went to follow up the
papers, he found them still in the Quezon office and so he personally brought
the papers to the Manila office of BF Lifeman Insurance Corporation. It was only
on November 27, 1987 that said papers were received in Manila.
Without knowing that Perez died on November 25, 1987, BF Lifeman Insurance
Corporation approved the application and issued the corresponding policy for
the P50,000.00 on December 2, 1987
Virginia went to Manila to claim the benefits under the insurance policies of the
deceased. She was paid P40,000.00 under the first insurance policy for
P20,000.00 (double indemnity in case of accident) but the insurance company
refused to pay the claim under the additional policy coverage of P50,000.00, the
proceeds of which amount to P150,000.00 in view of a triple indemnity rider on
the insurance policy.
In its letter of January 29, 1988 to Virginia A. Perez, the insurance company
maintained that the insurance for P50,000.00 had not been perfected at the
time of the death of Primitivo Perez. Consequently, the insurance company
refunded the amount of P2,075.00 which Virginia Perez had paid
Lifeman filed for the rescission and the declaration of nullity. Perez, on the
other hand, averred that the deceased had fulfilled all his prestations under the
contract and all the elements of a valid contract are present.
RTC ruled in favor of Perez. CA reversed.

Issue: W/N there was a perfected additional insurance contract.
Held: The contract was not perfected.
Insurance is a contract whereby, for a stipulated consideration, one party
undertakes to compensate the other for loss on a specified subject by specified
perils. A contract, on the other hand, is a meeting of the minds between two
persons whereby one binds himself, with respect to the other to give something
or to render some service.
Consent must be manifested by the meeting of the offer and the
acceptance upon the thing and the cause which are to constitute the contract.
The offer must be certain and the acceptance absolute. When Primitivo filed an
application for insurance, paid P2,075.00 and submitted the results of his
medical examination, his application was subject to the acceptance of private
respondent BF Lifeman Insurance Corporation. The perfection of the contract of
insurance between the deceased and respondent corporation was further
conditioned upon compliance with the following requisites stated in the
application form:
"there shall be no contract of insurance unless and until a policy is
issued on this application and that the said policy shall not take effect
until the premium has been paid and the policy delivered to and
accepted by me/us in person while I/We, am/are in good health."
The assent of private respondent BF Lifeman Insurance Corporation
therefore was not given when it merely received the application form and all the
requisite supporting papers of the applicant. Its assent was given when it issues
a corresponding policy to the applicant. Under the abovementioned provision, it
is only when the applicant pays the premium and receives and accepts the
policy while he is in good health that the contract of insurance is deemed to
have been perfected.
It is not disputed, however, that when Primitivo died on November 25,
1987, his application papers for additional insurance coverage were still with the
branch office of respondent corporation in Gumaca and it was only two days
later, or on November 27, 1987, when Lalog personally delivered the application
papers to the head office in Manila. Consequently, there was absolutely no way
the acceptance of the application could have been communicated to the
applicant for the latter to accept inasmuch as the applicant at the time was
already dead.
CIR v. Lincoln Phil Life 379 SCRA 423 (2002)
Facts:

!"#$%&"'( * &++,- .$!/#0" 2:
Anbochi, Atillo, Weigand
Taken from Rhys Alexeis reviewer. Digests from upperbatch.

In the years prior to 1984, Lincoln issued a special kind of life insurance policy
known as the "Junior Estate Builder Policy," the distinguishing feature of which
is a clause providing for an automatic increase in the amount of life insurance
coverage upon attainment of a certain age by the insured without the need of
issuing a new policy. The clause was to take effect in the year 1984.
Documentary stamp taxes due on the policy were paid to the petitioner only on
the initial sum assured.
Subsequently, petitioner issued deficiency documentary stamps tax assessment
for the year 1984, corresponding to the amount of automatic increase of the
sum assured on the policy issued by respondent.
Lincoln questioned the deficiency assessments and sought their cancellation in
a petition filed in the Court of Tax Appeals. CTA found no basis for the
assessment. CA affirmed.

Issue: W/N the automatic increase of the sum assured on the policy is taxable.
Held: YES.
CIR claims that the "automatic increase clause" in the subject insurance
policy is separate and distinct from the main agreement and involves another
transaction; and that, while no new policy was issued, the original policy was
essentially re-issued when the additional obligation was assumed upon the
effectivity of this "automatic increase clause" in 1984; hence, a deficiency
assessment based on the additional insurance not covered in the main policy is
in order. The SC agreed with this contention.
The subject insurance policy at the time it was issued contained an
"automatic increase clause." Although the clause was to take effect only in
1984, it was written into the policy at the time of its issuance. The distinctive
feature of the "junior estate builder policy" called the "automatic increase
clause" already formed part and parcel of the insurance contract, hence, there
was no need for an execution of a separate agreement for the increase in the
coverage that took effect in 1984 when the assured reached a certain age.
It is clear from Section 173 of the NIRC that the payment of documentary
stamp taxes is done at the time the act is done or transaction had and the tax
base for the computation of documentary stamp taxes on life insurance policies
under Section 183 of NIRC is the amount fixed in policy, unless the interest of a
person insured is susceptible of exact pecuniary measurement.
Logically, we believe that the amount fixed in the policy is the figure written
on its face and whatever increases will take effect in the future by reason of the
"automatic increase clause" embodied in the policy without the need of another
contract.
Here, although the automatic increase in the amount of life insurance
coverage was to take effect later on, the date of its effectivity, as well as the
amount of the increase, was already definite at the time of the issuance of the
policy. Thus, the amount insured by the policy at the time of its issuance
necessarily included the additional sum covered by the automatic increase
clause because it was already determinable at the time the transaction was
entered into and formed part of the policy.
The "automatic increase clause" in the policy is in the nature of a
conditional obligation under Article 1181, 8 by which the increase of the
insurance coverage shall depend upon the happening of the event which
constitutes the obligation. In the instant case, the additional insurance that took
effect in 1984 was an obligation subject to a suspensive obligation, 9 but still a
part of the insurance sold to which private respondent was liable for the
payment of the documentary stamp tax.
Grepalife v. CA 89 SCRA 543
Facts:
On March 14, 1957, respondent Ngo Hing filed an application with Grepalife for
a 20-yr endowment policy for 50T on the life of his one year old daughter Helen
Go.
All the essential data regarding Helen was supplied by Ngo to Lapu-Lapu
Mondragon, the branch manager of Grepalife-Cebu. Mondragon then typed the
data on the application form which was later signed by Ngo.
Ngo then paid the insurance premium and a binding deposit receipt was issued
to him. The binding receipt contained the following provision: If the applicant
shall not have been insurable xxx and the Company declines to approve the
application, the insurance applied for shall not have been in force at any time
and the sum paid shall be returned to the applicant upon the surrender of this
receipt.
Mondragon wrote on the bottom of the application form his strong
recommendation for the approval of the insurance application.
On Apr 30, 1957, Mondragon received a letter from Grepalife Main office
disapproving the insurance application of Ngo for the simple reason that the
20yr endowment plan is not available for minors below 7 yrs old.
Mondragon wrote back the main office again strongly recommending the
approval of the endowment plan on the life of Helen, adding that Grepalife was
the only insurance company NOT selling endowment plans to children.

!"#$%&"'( * &++,- .$!/#0" 21
Anbochi, Atillo, Weigand
Taken from Rhys Alexeis reviewer. Digests from upperbatch.

On may 1957, Helen died of influenza with complication of broncho pneumonia.
Ngo filed a claim with Gepalife, but the latter denied liability on the ground that
there was no contract between the insurer and the insured and a binding receipt
is NOT evidence of such contract.

Issue: W/N the binding deposit receipt, constituted a temporary contract of life
insurance.
Held: NO.
The binding receipt in question was merely an acknowledgement on behalf
of the company, that the latters branch office had received from the applicant,
the insurance premium and had accepted the application subject for processing
by the insurance company, and that the latter will either approve or reject the
same on the basis of whether or not the applicant is insurable on standard
rates.
Since Grepalife disapproved the insurance application of Ngo, the binding
deposit receipt had never became on force at any time, pursuant to par. E of the
said receipt. A binding receipt is manifestly merely conditional and does NOT
insure outright. Where an agreement is made between the applicant and the
agent, NO liability shall attach until the principal approves the risk and a receipt
is given by the agent.
The acceptance is merely conditional, and is subordinated to the act of the
company in approving or rejecting the application. Thus in life insurance, a
binding slip or binding receipt does NOT insure by itself.
Pacific Timber v. CA 112 SCRA 199
Facts:
On March 13, 1963, Pacific secured temporary insurance from the Workemens
Insurance Co. for its exportation of logs to Japan. Workmen issued on said
date Cover Note 1010 insuring said cargo.
The regular marine policies were issued by the company in favor of Pacific on
Apr 2, 1963. The 2 marine policies bore the number 53H01032 and 53H01033.
After the issuance of the cover note but BEFORE the issuance of the 2 policies,
some of the logs intended to be exported were lost due to a typhoon.
Pacific filed its claim with the company, but the latter refused, contending that
said loss may not be considered as covered under the cover note because such
became null and void by virtue of the issuance of the marine policies.

Issue: W/N the cover not was without consideration, thus null and void.
Held: It was with consideration.
SC upheld Pacifics contention that said cover not was with consideration.
The fact that no separate premium was paid on the cover note before the loss
was insured against occurred does not militate against the validity of Pacifics
contention, for no such premium could have been paid, since by the nature of
the cover note, it did not contain, as all cover notes do not contain, particulars of
the shipment that would serve as basis for the computation of the premiums.
As a logical consequence, no separate premiums are required to be paid on a
cover note.

If the note is to be treated as a separate policy instead of integrating it to
the regular policies subsequently issued, its purpose would be meaningless for
it is in a real sense a contract, not a mere application.
Bonifacio Bros. v. Mora 20 SCRA 262
Facts:
Enrique Mora mortgaged his Odlsmobile sedan car to HS Reyes Inc. with the
condition that Mora would insure the car with HS Reyes as beneficiary.
The car was then insured with State Insurance Company and the policy
delivered to Mora.
During the effectivity of the insurance contract, the car figured in an accident.
The company then assigned the accident to an insurance appraiser for
investigation and appraisal of the damage.
Mora without the knowledge and consent of HS Reyes, authorized Bonifacio
Bros to fix the car, using materials supplied by the Ayala Auto Parts Company.
For the cost of Labor and materials, Mora was billed P2,102.73. The bill was
sent to the insurers appraiser. The insurance company drew a check in the
amount of the insurance proceeds and entrusted the check to its appraiser for
delivery to the proper party.
The car was delivered to Mora without the consent of HS Reyes, and without
payment to Bonifacio Bros and Ayala.
Upon the theory that the insurance proceeds should be directly paid to them,
Bonifacio and Ayala filed a complaint against Mora and the insurer with the
municipal court for the collection of P2,102.73.

!"#$%&"'( * &++,- .$!/#0" 22
Anbochi, Atillo, Weigand
Taken from Rhys Alexeis reviewer. Digests from upperbatch.

The insurance company filed its answer with a counterclaim for interpleader,
requiring Bonifacio and HS Reyes to interplead in order to determine who has a
better right to the proceeds.

Issue: W/N there is privity of contract between Bonficacio and Ayala on one
hand and State Insurance on the other.
Held: NONE.
It is fundamental that contracts take effect only between the parties thereto,
except in some specific instance provided by law where the contract contains
some stipulation in favor of a third person. Such stipulation is known as a
stipulation pour autrui; or a provision in favor of a third person not a party to the
contract.

Under this doctrine, a third person is allowed to avail himself of a benefit
granted to him by the terms of the contract, provided that the contracting parties
have clearly and deliberately conferred a favor upon such person.
Consequently, a third person NOT a party to the contract has NO action against
the aprties thereto, and cannot generally demand the enforcement of the same.
The question of whether a third person has an enforceable interest in a
contract must be settled by determining whether the contracting parties
intended to tender him such an interest by deliberately inserting terms in their
agreement with the avowed purpose of conferring favor upon such third person.
IN this connection, this court has laid down the rule that the fairest test to
determine whether the interest of a 3
rd
person in a contract is a stipulation pour
autrui or merely an incidental interest, is to rely upon the intention of the parties
as disclosed by their contract.
In the instant case the insurance contract does not contain any words or
clauses to disclose an intent to give any benefit to any repairmen or material
men in case of repair of the car in question. The parties to the insurance
contract omitted such stipulation, which is a circumstance that supports the said
conclusion. On the other hand, the "loss payable" clause of the insurance policy
stipulates that "Loss, if any, is payable to H.S. Reyes, Inc." indicating that it was
only the H.S. Reyes, Inc. which they intended to benefit.
A policy of insurance is a distinct and independent contract between the
insured and insurer, and third persons have no right either in a court of equity,
or in a court of law, to the proceeds of it, unless there be some contract of trust,
expressed or implied, by the insured and third person. In this case, no contract
of trust, express or implied. In this case, no contract of trust, expressed or
implied exists. We, therefore, agree with the trial court that no cause of action
exists in favor of the appellants in so far as the proceeds of insurance are
concerned. The appellant's claim, if at all, is merely equitable in nature and
must be made effective through Enrique Mora who entered into a contract with
the Bonifacio Bros Inc. This conclusion is deducible not only from the principle
governing the operation and effect of insurance contracts in general, but is
clearly covered by the express provisions of section 50 of the Insurance Act
(now Sec. 53).
The policy in question has been so framed that "Loss, if any, is payable to
H. S. Reyes, Inc." which unmistakably shows the intention of the parties.

Coquia v. Fieldmens Insurance 26 SCRA 172

Facts:
On Dec. 1, 1961, Fieldmens Insurance co. Issued in favor of the Manila Yellow
Taxicab a common carrier insurance policy with a stipulation that the company
shall indemnify the insured of the sums which the latter wmy be held liable for
with respect to death or bodily injury to any faire-paying passenger including
the driver and conductor.
The policy also stated that in the event of the death of the driver, the Company
shall indemnify his personal representatives and at the Companys option may
make indemnity payable directly to the claimants or heirs of the claimants.
During the policys lifetime, a taxicab of the insured driven by Coquia met an
accident and Coquia died.
When the company refused to pay the only heirs of Coquia, his parents, they
institued this complaint. The company contends that plaintiffs have no cause of
action since the Coquias have no contractual relationship with the company.

Issue: W/N plaintiffs have the right to collect on the policy.
Held: YES.

!"#$%&"'( * &++,- .$!/#0" 23
Anbochi, Atillo, Weigand
Taken from Rhys Alexeis reviewer. Digests from upperbatch.

Athough, in general, only parties to a contract may bring an action based
thereon, this rule is subject to exceptions, one of which is found in the second
paragraph of Article 1311 of the Civil Code of the Philippines, reading: "If a
contract should contain some stipulation in favor of a third person, he may
demand its fulfillment provided he communicated his acceptance to the obligor
before its revocation. A mere incidental benefit or interest of a person is not
sufficient. The contracting parties must have clearly and deliberately conferred a
favor upon a third person." This is but the restatement of a well-known principle
concerning contracts pour autrui, the enforcement of which may be demanded
by a third party for whose benefit it was made, although not a party to the
contract, before the stipulation in his favor has been revoked by the contracting
parties
In the case at bar, the policy under consideration is typical of contracts pour
autrui this character being made more manifest by the fact that the deceased
driver paid fifty percent (50%) of the corresponding premiums, which were
deducted from his weekly commissions. Under these conditions, it is clear that
the Coquias who, admittedly, are the sole heirs of the deceased have a
direct cause of action against the Company, and, since they could have
maintained this action by themselves, without the assistance of the insured it
goes without saying that they could and did properly join the latter in filing the
complaint herein.
Del Val v. Del Val 29 Phil 535
Facts:
Petitioners and private respondents are brothers and Sisters and are the only
heirs and next of kin of Gregorio del Val who died intestate.
It was found out that the deceased took out insurance on his life for the sum of
40T and made it payable to private respondents as sole beneficiary.
After Gregorios death, Andres collected the proceeds of the policy.
Of the said policy, Andres paid 18T to redeem some real property which
Gregorio had sold to third persons during his lifetime.
Said redemption of the property was made by Andres laywer in the name of
Andres and the petitioners. (Accdg to Andres, said redemption in the name of
Petitioners and himself was without his knowledge and that since the
redemption, petitioners have been in possession of the property)
Petitioners now contend that the amount of the insurance policy belonged to the
estate of the deceased and not to Andres personally.
Pet filed a complaint for partition of property including the insurance proceeds
Andress claims that he is the sole owner of the proceeds and prayed that he be
declared:
Sole owner of the real property, redeemed with the use of the insurance
proceeds and its remainder;
Petitioners to account for the use and occupation of the premises.

Issue: W/N the petitioners have a right to the insurance proceeds?
Held: NOPE.
The contract of life insurance is a special contract and the destination of the
proceeds thereof is determined by special laws which deal exclusively with the
subject. Our civil code has no provisions which relate directly and specifically to
life-insurance contracts of to the destination of life-insurance proceeds that
subject is regulated exclusively by the Code of Commerce. Thus, contention of
petitioners that proceeds should be considered as a dontation or gift and should
be included in the estate of the deceased is UNTENABLE.

Since the repurchase has been made n the names of all the heirs instead
of the defendant alone, petitioners claim that the property belongs to the heirs in
common and not to the defendant alone. The SC held that if it is established by
evidence that that was his intention and that the real estate was delivered to the
plaintiffs with that understanding, then it is probable that their contention is
correct and that they are entitled to share equally with the defendant.
HOWEVER, it appears from the evidence that the conveyances were taken in
the name of the plaintiffs without the knowledge and consent of Andres, or that
it was not his intention to make a gift to them of real estate, when it belongs to
him.
RCBC v. CA 289 SCRA 292 (1998)
Facts:
GOYU applied for credit facilities and accommodations with RCBC. After due
evaluation, a credit facility in the amount of P30 million was initially granted.
Upon GOYU's application increased GOYU's credit facility to P50 million, then
to P90 million, and finally to P117 million
As security for its credit facilities with RCBC, GOYU executed two REM and two
CM in favor of RCBC, which were registered with the Registry of Deeds at.
Under each of these four mortgage contracts, GOYU committed itself to insure
the mortgaged property with an insurance company approved by RCBC, and
subsequently, to endorse and deliver the insurance policies to RCBC.

!"#$%&"'( * &++,- .$!/#0" 24
Anbochi, Atillo, Weigand
Taken from Rhys Alexeis reviewer. Digests from upperbatch.

GOYU obtained in its name a total of 10 insurance policies from MICO. In
February 1992, Alchester Insurance Agency, Inc., the insurance agent where
GOYU obtained the Malayan insurance policies, issued nine endorsements in
favor of RCBC seemingly upon instructions of GOYU
On April 27, 1992, one of GOYU's factory buildings in Valenzuela was gutted by
fire. Consequently, GOYU submitted its claim for indemnity.
MICO denied the claim on the ground that the insurance policies were either
attached pursuant to writs of attachments/garnishments issued by various
courts or that the insurance proceeds were also claimed by other creditors of
GOYU alleging better rights to the proceeds than the insured.
GOYU filed a complaint for specific performance and damages. RCBC, one of
GOYU's creditors, also filed with MICO its formal claim over the proceeds of the
insurance policies, but said claims were also denied for the same reasons that
AGCO denied GOYU's claims.
However, because the endorsements do not bear the signature of any officer of
GOYU, the trial court, as well as the Court of Appeals, concluded that the
endorsements are defective and held that RCBC has no right over the
insurance proceeds.

Issue: W/N RCBC has a right over the insurance proceeds.
Held: RCBC has a right over the insurance proceeds.
It is settled that a mortgagor and a mortgagee have separate and distinct
insurable interests in the same mortgaged property, such that each one of them
may insure the same property for his own sole benefit. There is no question that
GOYU could insure the mortgaged property for its own exclusive benefit. In the
present case, although it appears that GOYU obtained the subject insurance
policies naming itself as the sole payee, the intentions of the parties as shown
by their contemporaneous acts, must be given due consideration in order to
better serve the interest of justice and equity.
It is to be noted that 9 endorsement documents were prepared by Alchester
in favor of RCBC. The Court is in a quandary how Alchester could arrive at the
idea of endorsing any specific insurance policy in favor of any particular
beneficiary or payee other than the insured had not such named payee or
beneficiary been specifically disclosed by the insured itself. It is also significant
that GOYU voluntarily and purposely took the insurance policies from MICO, a
sister company of RCBC, and not just from any other insurance company.
Alchester would not have found out that the subject pieces of property were
mortgaged to RCBC had not such information been voluntarily disclosed by
GOYU itself. Had it not been for GOYU, Alchester would not have known of
GOYU's intention of obtaining insurance coverage in compliance with its
undertaking in the mortgage contracts with RCBC, and verify, Alchester would
not have endorsed the policies to RCBC had it not been so directed by GOYU.
On equitable principles, particularly on the ground of estoppel, the Court is
constrained to rule in favor of mortgagor RCBC. RCBC, in good faith, relied
upon the endorsement documents sent to it as this was only pursuant to the
stipulation in the mortgage contracts. We find such reliance to be justified under
the circumstances of the case. GOYU failed to seasonably repudiate the
authority of the person or persons who prepared such endorsements. Over and
above this, GOYU continued, in the meantime, to enjoy the benefits of the credit
facilities extended to it by RCBC. After the occurrence of the loss insured
against, it was too late for GOYU to disown the endorsements for any imagined
or contrived lack of authority of Alchester to prepare and issue said
endorsements. If there had not been actually an implied ratification of said
endorsements by virtue of GOYU's inaction in this case, GOYU is at the very
least estopped from assailing their operative effects.

To permit GOYU to capitalize on its non-confirmation of these
endorsements while it continued to enjoy the benefits of the credit facilities of
RCBC which believed in good faith that there was due endorsement pursuant to
their mortgage contracts, is to countenance grave contravention of public policy,
fair dealing, good faith, and justice. Such an unjust situation, the Court cannot
sanction. Under the peculiar circumstances obtaining in this case, the Court is
bound to recognize RCBC's right to the proceeds of the insurance policies if not
for the actual endorsement of the policies, at least on the basis of the equitable
principle of estoppel.
GOYU cannot seek relief under Section 53 of the Insurance Code which
provides that the proceeds of insurance shall exclusively apply to the interest of
the person in whose name or for whose benefit it is made. The peculiarity of the
circumstances obtaining in the instant case presents a justification to take
exception to the strict application of said provision, it having been sufficiently
established that it was the intention of the parties to designate RCBC as the
party for whose benefit the insurance policies were taken out. Consider thus the
following:
1. It is undisputed that the insured pieces of property were the subject of
mortgage contracts entered into between RCBC and GOYU in
consideration of and for securing GOYU's credit facilities from RCBC.
The mortgage contracts contained common provisions whereby

!"#$%&"'( * &++,- .$!/#0" 25
Anbochi, Atillo, Weigand
Taken from Rhys Alexeis reviewer. Digests from upperbatch.

GOYU, as mortgagor, undertook to have the mortgaged property
properly covered against any loss by an insurance company
acceptable to RCBC.
2. GOYU voluntarily procured insurance policies to cover the mortgaged
property from MICO, no less than a sister company of RCBC and
definitely an acceptable insurance company to RCBC.
3. Endorsement documents were prepared by MICO's underwriter,
Alchester Insurance Agency, Inc., and copies thereof were sent to
GOYU, MICO and RCBC. GOYU did not assail, until of late, the
validity of said endorsements.
4. GOYU continued until the occurrence of the fire, to enjoy the benefits
of the credit facilities extended by RCBC which was conditioned upon
the endorsement of the insurance policies to be taken by GOYU to
cover the mortgaged properties.

This Court can not over stress the fact that upon receiving its copies of the
endorsement documents prepared by Alchester, GOYU, despite the absence
written conformity thereto, obviously considered said endorsement to be
sufficient compliance with its obligation under the mortgage contracts since
RCBC accordingly continued to extend the benefits of its credit facilities and
GOYU continued to benefit therefrom. Just as plain too is the intention of the
parties to constitute RCBC as the beneficiary of the various insurance policies
obtained by GOYU. The intention of the parties will have to be given full force
and effect in this particular case. The insurance proceeds may, therefore, be
exclusively applied to RCBC, which under the factual circumstances of the
case, is truly the person or entity for whose benefit the policies were clearly
intended.
SSS v Davac
Facts:

Petronillo became a member of the SSS and assigned Candelaria as his
beneficiary. He later on died. It was found out that two women were alleging to
be the wife of Petronillo. One was Lourdes and another was Candelaria who
was the designated beneficiary.

As there was contesting claims, SSS filed an interpleader.

Held: The benefits accruing membership in the SSS do not form part of the
properties of the conjugal partnership of the covered member. They are
disbursed from a public special fund created by Congress in pursuance to the
declared policy of the Republic to develop, establish gradually and perfect a
social security system which, shall provide protection against the hazards of
disability, sickness, old age and death.
The sources of this special fund are the covered employees contribution
(equivalent to 2"percent of the employee's monthly compensation, the
employers contribution (equivalent to 3% of the monthly compensation of the
covered employee) and the Government contribution which consists in yearly
appropriation of public funds to assure the maintenance of an adequate working
balance of the System. Additionally, Sec. 21 of the Social Security Act provides
that the benefits in the Act shall not be diminished, and to guarantee such
benefits, the Government accepts general responsibility for the solvency of the
System.
As such, the benefits under the Act is in the nature of a privilege or an
arrangement secured by law, pursuant to the policy of the State to provide
social security to the workingmen. The amounts to be received cannot be
considered as property earned, by the member during his lifetime. His
contribution to the fund, it must be noted, constitutes only an insignificant
portion thereof. Then, the benefits specifically declared not transferable, and
exempted form taxes, legal processes, and liens. Furthermore, in the settlement
of claims thereunder the procedure to be observed is governed not by the
general provisions of law, but by the rules and regulations promulgated by the
Commission. Thus, if the money is payable to the estate of a deceased
member, it is the commission, not the probate or regular court that determines
the person/s to whom it is payable. That the benefits payable under the Act are
not intended to form part of the estate of the covered members may be
gathered from a subsequent amendment, which provides for the non-
transferability of the benefit, and the benefits shall only be paid only to the
persons entitled thereto in accordance with the provisions of the Act.
Thus, if there is a named beneficiary, and the designation is not invalid, it is not
the heirs of the employee who are entitled to receive the benefits, unless they
are designated beneficiaries themselves. It is only when there is no designated
beneficiaries or when the designation is void, that the laws of succession are
applicable. And the Social Security Act is not a law of succession.
In Re: Mario Changlionco
Facts:

!"#$%&"'( * &++,- .$!/#0" 26
Anbochi, Atillo, Weigand
Taken from Rhys Alexeis reviewer. Digests from upperbatch.

A claim for retirement benefits was filed by the heirs of Mario, an attorney. Such
was claim was filed under RA No. 1616, as amended by RA No. 4986, which
was approved by the Court. At the time of the death of Mario, he was 63 years
old, with more than 38 years of service in the government. He did not have any
pending criminal, administrative or any other case against him, and neither did
he have any money or property for which he was accountable. The highest
salary he received was PhP 18,700 pa. Claims by his heirs were filed for the
deceaseds benefits under the GSIS. Aside from his widow, Dra. Fidel
Chanliongco, and an only Intimate, it appears that there are other heirs which
includes Angelina and Mario Jr., born out of wedlock.
According to the law, the benefits accruing to the deceased consists of: 1)
retirement benefits; 2) money value of terminal leave; 3) life insurance and 4)
refund of retirement premium.
Held:
The record also shows that the late Atty. Mario died ab intestato and that he
filed or turned over to the state in his application for membership with the GSIS
the beneficiary or retirement benefits, should he die before retirement. Hence,
the retirement benefits shall accrue to his estate and will be distributed among
his legal heirs in with the benefits on intestate succession, as in the caw of a fife
if no benefit is named in the policy.
Vda. De Consuegra v GSIS
Facts:
The late Jose Consuegra, at the time of his death, was employed as a shop
foreman of the office of the District Engineer in the province of Surigao del
Norte. In his lifetime, he contracted 2 marriages, the first with respondent,
Rosario Diaz, out of which marriage, were born 2 children, Jose, Jr., and Pedro,
both of which predeceased their father; and the second marriage which was
contracted in good faith which the first marriage was subsisting. The second
marriage was with Basilia Berdin, out of which marriage were born seven
children, namely, Juliana, Pacita, Maria Lourdes, Jose, Rodrigo, Lenida and
Luz, all surnamed Consuegra.
Being a member of the GSIS, the proceeds of his life insurance were paid by
the GSIS to petitioner Berdin and her children who were named as beneficiaries
named in the policy. Having been in the service of the government for 22.5
years, Consuegra was entitled to retirement insurance benefits in the sum of
Php6,304.47 pursuant to CA 186, amended by RAs 1616 and 3836. Consuegra
did not designate any beneficiary who would receive the retirement insurance
benefits due to him.
Held:
If Consuegra had 22.5 years of service in the government when he died, it
follows that he started in the government service sometime during the early part
of 1943. In 1943, CA 186 was not yet amended, and the only benefits then
provided for in the said Act were those that proceeded from a life insurance.
Upon entering the government service, Consuegra became a compulsory
member of GSIS, being automatically insured on his life, pursuant to the Act
then in force. During 1943, the operation of the GSIS was suspended because
of war, and the operation was resumed sometime in 1946. When Consuegra
designated his beneficiaries in his life insurance he could not have intended
those beneficiaries of his life insurance as also the beneficiaries of his
retirement insurance because the provisions on retirement insurance under the
GSIS came about only when CA 186 was amended by RA 660. Hence, it
cannot be said that becase the appellants designated beneficiaries in
Consuegras life insurance, they automatically became the beneficiaries also of
his retirement insurance.
In the case of the proceeds of a life insurance, the same are paid to whoever is
named in the beneficiary in the life insurance policy. As in case of a life
insurance provided for in the Insurance Act, the beneficiary in a life insurance
may designate any person as beneficiary in a life insurance under the GSIS
may not necessarily be an heir of the insured. The insured in a life insurance
may designate any person as beneficiary unless disqualified to be so under the
provisions of the Civil Code. And in the absence of any beneficiary named in the
life insurance policy, the proceeds of the insurance will go to the estate of the
insured.
Retirement insurance is primarily intended for the benefit of the employee to
provide for his old age, or incapacity, after rendering service in the government
for a required number of years. If the employee reaches the age of retirement,
he gets the retirement benefits even to the exclusion of the
beneficiary/beneficiaries named in his application for retirement insurance. The
beneficiary of the retirement insurance can only claim the process of the
retirement insurance if the employee dies before retirement. If the employee
failed or overlooked to state the beneficiary of his retirement insurance, the
retirement benefits will accrue to his estate and will be given to his legal heirs in
accordance with law, as in the case of the life insurance if no beneficiary is
named in the insurance policy.

!"#$%&"'( * &++,- .$!/#0" 27
Anbochi, Atillo, Weigand
Taken from Rhys Alexeis reviewer. Digests from upperbatch.

GSIS had correctly acted when it ruled that the proceeds of the retirement
insurance of the late Jose Consuegra should be divided equally between his
first wife and Berdin and his children by her, on the other; and the lower court
did not commit error when it confirmed the action of GSIS, it being accepted as
a fact that the second marriage of Jose Consuegra to Berdin was contracted in
good faith.

Heirs of Maramag v Maramag 588 SCRA 744
Facts: The case stems from a petition3 filed against respondents with the
Regional Trial Court, Branch 29, for revocation and/or reduction of insurance
proceeds for being void and/or inofficious, with prayer for a temporary
restraining order (TRO) and a writ of preliminary injunction.
The petition alleged that: (1) petitioners were the legitimate wife and children of
Loreto Maramag (Loreto), while respondents were Loretos illegitimate family;
(2) Eva de Guzman Maramag (Eva) was a concubine of Loreto and a suspect in
the killing of the latter, thus, she is disqualified to receive any proceeds from his
insurance policies from Insular Life Assurance Company, Ltd. (Insular) and
Great Pacific Life Assurance Corporation (Grepalife); (3) the illegitimate children
of LoretoOdessa, Karl Brian, and Trisha Angeliewere entitled only to one-
half of the legitime of the legitimate children, thus, the proceeds released to
Odessa and those to be released to Karl Brian and Trisha Angelie were
inofficious and should be reduced; and (4) petitioners could not be deprived of
their legitimes, which should be satisfied first. In support of the prayer for TRO
and writ of preliminary injunction, petitioners alleged, among others, that part of
the insurance proceeds had already been released in favor of Odessa, while the
rest of the proceeds are to be released in favor of Karl Brian and Trisha Angelie,
both minors, upon the appointment of their legal guardian. Petitioners also
prayed for the total amount of P320,000.00 as actual litigation expenses and
attorneys fees.
Insular admitted that Loreto misrepresented Eva as his legitimate wife and
Odessa, Karl Brian, and Trisha Angelie as his legitimate children, and that they
filed their claims for the insurance proceeds of the insurance policies; that when
it ascertained that Eva was not the legal wife of Loreto, it disqualified her as a
beneficiary and divided the proceeds among Odessa, Karl Brian, and Trisha
Angelie, as the remaining designated beneficiaries; and that it released
Odessas share as she was of age, but withheld the release of the shares of
minors Karl Brian and Trisha Angelie pending submission of letters of
guardianship. Insular alleged that the complaint or petition failed to state a
cause of action insofar as it sought to declare as void the designation of Eva as
beneficiary, because Loreto revoked her designation as such in Policy No.
A001544070 and it disqualified her in Policy No. A001693029; and insofar as it
sought to declare as inofficious the shares of Odessa, Karl Brian, and Trisha
Angelie, considering that no settlement of Loretos estate had been filed nor had
the respective shares of the heirs been determined. Insular further claimed that
it was bound to honor the insurance policies designating the children of Loreto
with Eva as beneficiaries pursuant to Section 53 of the Insurance Code.
In its own answer with compulsory counterclaim, Grepalife alleged that Eva was
not designated as an insurance policy beneficiary; that the claims filed by
Odessa, Karl Brian, and Trisha Angelie were denied because Loreto was
ineligible for insurance due to a misrepresentation in his application form that he
was born on December 10, 1936 and, thus, not more than 65 years old when he
signed it in September 2001; that the case was premature, there being no claim
filed by the legitimate family of Loreto; and that the law on succession does not
apply where the designation of insurance beneficiaries is clear.
Issue: (A)re the members of the legitimate family entitled to the proceeds of the
insurance for the concubine?
Held: In this case, it is clear from the petition filed before the trial court that,
although petitioners are the legitimate heirs of Loreto, they were not named as
beneficiaries in the insurance policies issued by Insular and Grepalife. The
basis of petitioners claim is that Eva, being a concubine of Loreto and a
suspect in his murder, is disqualified from being designated as beneficiary of
the insurance policies, and that Evas children with Loreto, being illegitimate
children, are entitled to a lesser share of the proceeds of the policies. They also
argued that pursuant to Section 12 of the Insurance Code,19 Evas share in the
proceeds should be forfeited in their favor, the former having brought about the
death of Loreto. Thus, they prayed that the share of Eva and portions of the
shares of Loretos illegitimate children should be awarded to them, being the
legitimate heirs of Loreto entitled to their respective legitimes.
It is evident from the face of the complaint that petitioners are not entitled to a
favorable judgment in light of Article 2011 of the Civil Code which expressly
provides that insurance contracts shall be governed by special laws, i.e., the
Insurance Code. Section 53 of the Insurance Code states


!"#$%&"'( * &++,- .$!/#0" 28
Anbochi, Atillo, Weigand
Taken from Rhys Alexeis reviewer. Digests from upperbatch.

SECTION 53. The insurance proceeds shall be applied exclusively to the proper
interest of the person in whose name or for whose benefit it is made unless
otherwise specified in the policy.
Pursuant thereto, it is obvious that the only persons entitled to claim the
insurance proceeds are either the insured, if still alive; or the beneficiary, if the
insured is already deceased, upon the maturation of the policy. The exception
to this rule is a situation where the insurance contract was intended to benefit
third persons who are not parties to the same in the form of favorable
stipulations or indemnity. In such a case, third parties may directly sue and
claim from the insurer.
Petitioners are third parties to the insurance contracts with Insular and Grepalife
and, thus, are not entitled to the proceeds thereof. Accordingly, respondents
Insular and Grepalife have no legal obligation to turn over the insurance
proceeds to petitioners. The revocation of Eva as a beneficiary in one policy
and her disqualification as such in another are of no moment considering that
the designation of the illegitimate children as beneficiaries in Loretos insurance
policies remains valid. Because no legal proscription exists in naming as
beneficiaries the children of illicit relationships by the insured,22 the shares of
Eva in the insurance proceeds, whether forfeited by the court in view of the
prohibition on donations under Article 739 of the Civil Code or by the insurers
themselves for reasons based on the insurance contracts, must be awarded to
the said illegitimate children, the designated beneficiaries, to the exclusion of
petitioners. It is only in cases where the insured has not designated any
beneficiary,23 or when the designated beneficiary is disqualified by law to
receive the proceeds,24 that the insurance policy proceeds shall redound to the
benefit of the estate of the insured.

San Miguel v. Law Union Rock 40 PHIL 674

Facts:
On Jan. 12, 1918, Dunn mortgaged a parcel of land to SMB to secure a debt of
10T.
Mortgage contract stated that Dunn was to have the property insured at his own
expense, authorizing SMB to choose the insurers and to receive the proceeds
thereof and retain so much of the proceeds as would cover the mortgage debt.
Dunn likewise authorized SMB to take out the insurance policy for him.
Brias, SMBs general manager, approached Law Union for insurance to the
extent of 15T upon the property. In the application, Brias stated that SMBs
interest in the property was merely that of a mortgagee.
Law Union, not wanting to issue a policy for the entire amount, issued one for
P7,500 and procured another policy of equal amount from Filipinas Cia de
Seguros. Both policies were issued in the name of SMB only and contained no
reference to any other interests in the propty. Both policies required
assignments to be approved and noted on the policy.
Premiums were paid by SMB and charged to Dunn. A year later, the policies
were renewed.
In 1917, Dunn sold the property to Harding, but no assignment of the policies
was made to the latter.
Property was destroyed by fire. SMB filed an action in court to recover on the
policies. Harding was made a defendant because by virtue of the sale, he
became the owner of the property, although the policies were issued in SMBs
name.
SMB sought to recover the proceeds to the extent of its mortgage credit with the
balance to go to Harding.
Insurance Companies contended that they were not liable to Harding because
their liability under the policies was limited to the insurable interests of SMB
only.
SMB eventually reached a settlement with the insurance companies and was
paid the balance of its mortgage credit. Harding was left to fend for himself.
Trial court ruled against Harding. Hence the appeal.

Issue: W/N the insurance companies are liable to Harding for the balance of
the proceeds of the 2 policies.
Held: NOPE.
Under the Insurance Act, the measure of insurable interest in the property
is the extent to which the insured might be daminified by the loss or injury
thereof. Also it is provided in the IA that the insurance shall be applied
exclusively to the proper interest of the person in whose name it is made.
Undoubtedly, SMB as the mortgagee of the property, had an insurable interest
therein; but it could NOT, an any event, recover upon the two policies an
amount in excess of its mortgage credit.
By virtue of the Insurance Act, neither Dunn nor Harding could have
recovered from the two policies. With respect to Harding, when he acquired the
property, no change or assignment of the policies had been undertaken. The

!"#$%&"'( * &++,- .$!/#0" 29
Anbochi, Atillo, Weigand
Taken from Rhys Alexeis reviewer. Digests from upperbatch.

policies might have been worded differently so as to protect the owner, but this
was not done.
If the wording had been: Payable to SMB, mortgagee, as its interests may
appear, remainder to whomsoever, during the continuance of the risk, may
become owner of the interest insured, it would have proved an intention to
insure the entire interest in the property, NOT merely SMBs and would have
shown to whom the money, in case of loss, should be paid. Unfortunately, this
was not what was stated in the policies.
If during the negotiation for the policies, the parties had agreed that even
the owners interest would be covered by the policies, and the policies had
inadvertently been written in the form in which they were eventually issued, the
lower court would have been able to order that the contract be reformed to give
effect to them in the sense that the parties intended to be bound. However,
there is no clear and satisfactory proof that the policies failed to reflect the real
agreement between the parties that would justify the reformation of these two
contracts.
A purchaser of insured property who does not take the precaution to obtain a
transfer of the policy of insurance cannot, in case of loss, recover upon such
contract, as the transfer of the property has the effect of suspending the
insurance until the purchaser becomes the owner of the policy as well as of the
property insured.
Development Insurance Corp. v IAC
Facts: A fire incurred in the building of the private respondent and it sued for
recovery of damages from the petitioner on the basis of an insurance contract
between them.
Held: The petitioner argues that since at the time of the fire, the building insured
was worth PhP 5.8 million, the private respondent should be considered its own
insurer for the difference between that amount and the face value of the policy
and should share pro rata in the loss sustained. Accordingly, the private
respondent is entitled to an indemnity of only PhP67,629.31, the rest of the loss
to be shouldered by it alone. However, there is no evidence on record that the
building was worth PhP5.8 million at the time of the loss; only the petitioner
says so and it does not back up its self-serving estimate with any independent
corroboration. On the contrary, the building was insured at PhP 2.5 million and
this must be considered, by agreement of the insurer and the insured, the actual
value of the property insured on the day the fire occurred. This valuation
becomes even more believable if it is remembered that at the time the building
was burned it was still under construction and not yet completed.
As defined under Sec. 60, an open policy is one in which the value of the thing
insured is not agreed upon but is left to be ascertained in case of loss. this
means that the actual loss, as determined, will represent the total indemnity due
the insured from the insurer except only that the total indemnity shall not exceed
the face value of the policy.
The actual loss has been ascertained in this case and, to repeat, this Court will
respect such factual determination in the absence of proof that it was arrived at
arbitrarily. There is no such showing. Hence, applying the open policy clause as
expressly agreed upon by the parties in their contract, the Court held that the
private respondent is entitled to the payment of indemnity under the said
contract in the total amount of PhP 508,867.00.
New Life Ent. v CA
Facts:
Julian Sy and Jose Sy have formed a business partnership named New Life
Enterprises and was engaged in the sale of construction materials. One of the
partners insured their stocks and was duly issued a fire insurance policy. A fire
broke out, destroying the stocks insured. They filed a claim for the proceeds of
the insurance, but was denied, on the allegation of breach of a condition, of
giving notice to the insurer of any insurance/s already effected, or subsequently
to be effected, covering the same property. Such notice is to be given and the
particulars thereof indorsed on the policy by or on behalf of the Company before
the occurrence of the loss, all benefits under the policy shall be deemed
forfeited, provided, that the condition shall not apply when the total insurance in
force at the time of the loss or damage is not more than PhP200,000. The trial
court decided in favor of New Life, but was reversed by the CA.
Held:
The terms of the contract are clear and unambiguous. The insured is
specifically required to disclose to the insurer any other insurance and its
particulars which he may have effected on the same subject matter. The
knowledge of such insurance by the insurers agent, even assuming the
acquisition thereof by the former, is not the notice that would estop the insurers
from denying the claim. Besides, the so-called theory of imputed knowledge,
that is, knowledge of the agent is knowledge of the principal, aside from being

!"#$%&"'( * &++,- .$!/#0" 3:
Anbochi, Atillo, Weigand
Taken from Rhys Alexeis reviewer. Digests from upperbatch.

of dubious applicability here was likewise been roundly refuted by respondent
court whose factual findings we find acceptable.
Thus, it points out that while the petitioner Julian Sy claimed that he had
informed insurance agent Alvarez regarding the co-insurance that he had
informed agent Alvarez regarding the co-insurance on the property, he
contradicted himself by inexplicably claiming that he had not read the terms of
the policies; that Chuan could not likewise have obtained such knowledge for
the same reason, aside from the fact that the insurance with Western was
obtained before those of Reliance and Equitable; and that the conclusion that
the trial court that Reliance and Equitable are sister companies is an unfounded
conjecture drawn from the mere fact that Chuan was an agent for both
companies which also had the same insurance claims adjuster. Availment of the
services of the same agents and adjusters by different companies is a common
practice in the insurance business and such facts do not warrant the speculative
conclusion of the trial court.
Furthermore, when the words and the language of documents are clear and
plain, or readily understandable, by an ordinary reader, there is absolutely no
room for interpretation. Courts are not allowed to make contracts for the parties;
rather, they will intervene only when the terms of the policy are ambiguous,
equivocal or uncertain. The parties must abide by the terms of the contract as it
constitutes the measure of the insurers liability and compliance therewith is a
condition precedent to the insureds right of recovery from the insurer.
While it is a cardinal principle of insurance law that a policy or contract of
insurance is to be construed liberally in favor of the insured and strictly against
the insurer, yet contracts of insurance, like other contracts, are to be construed
according to the sense and meaning of the terms which the parties themselves
have used. Moreover, the obligations arising from contracts have force and
effect of law, between the contracting parties and should be complied with in
good faith.
Petitioners should be aware of the fact that a party is not relieved of the duty to
exercise ordinary care and prudence that would be exacted in relation to other
contracts. The conformity of the insured to the terms of the policy is implied
from his failure to express any disagreement with what is provided for. It may be
true that the majority rule is that injured persons may accept the polices without
reading them, but, as an exception, as in the case that Sy was a businessman,
and the contract concerns indemnity in case of loss in his money-making trade
of which important considerations he could not have been unaware as it was
precisely the reason for his procuring the same.
Philamcare v CA
Facts: Ernani Trinos, deceased husband of Julita Trinos, applied for a health
care coverage with Philamcare Health Systems, Inc. In the standard application
form, he answered no to the following question: "Have you or any of your family
members ever consulted or been treated for high blood pressure, heart trouble,
diabetes, cancer, liver disease, asthma or peptic ulcer? (If Yes, give details). "
The application was approved for a period of one year from 1 March 1988 to 1
March 1989. Accordingly, he was issued Health Care Agreement P010194.
Under the agreement, Trinos' husband was entitled to avail of hospitalization
benefits, whether ordinary or emergency, listed therein. He was also entitled to
avail of "out-patient benefits" such as annual physical examinations, preventive
health care and other out-patient services. Upon the termination of the
agreement, the same was extended for another year from 1 March 1989 to 1
March 1990, then from 1 March 1990 to 1 June 1990. The amount of coverage
was increased to a maximum sum of P75,000.00 per disability. During the
period of his coverage, Ernani suffered a heart attack and was confined at the
Manila Medical Center (MMC) for one month beginning 9 March 1990. While
her husband was in the hospital, Trinos tried to claim the benefits under the
health care agreement. However, Philamcare denied her claim saying that the
Health Care Agreement was void. According to Philamcare, there was a
concealment regarding Ernani's medical history. Doctors at the MMC allegedly
discovered at the time of Ernani's confinement that he was hypertensive,
diabetic and asthmatic, contrary to his answer in the application form. Thus,
Trinos paid the hospitalization expenses herself, amounting to about
P76,000.00. After her husband was discharged from the MMC, he was attended
by a physical therapist at home. Later, he was admitted at the Chinese General
Hospital. Due to financial difficulties, however, Trinos brought her husband
home again. In the morning of 13 April 1990, Ernani had fever and was feeling
very weak. Trinos was constrained to bring him back to the Chinese General
Hospital where he died on the same day. On 24 July 1990, Trinos instituted with
the Regional Trial Court of Manila, Branch 44, an action for damages against
Philamcare and its president, Dr. Benito Reverente (Civil Case 90 53795). She
asked for reimbursement of her expenses plus moral damages and attorney's
fees. After trial, the lower court ruled against Philamcare and Reverente,
ordering them to pay and reimburse the medical and hospital coverage of the
late Ernani Trinos in the amount of P76,000.00 plus interest, until the amount is
fully paid to plaintiff who paid the same; the reduced amount of moral damages
of P10,000.00 to Trinos; the reduced amount of P10,000.00 as exemplary
damages to Trinos; and the attorney's fees of P20,000.00, plus costs of suit. On
appeal, the Court of Appeals affirmed the decision of the trial court but deleted
all awards for damages and absolved Reverente. Philamcare's motion for
reconsideration was denied. Hence, Philamcare brought the petition for review,

!"#$%&"'( * &++,- .$!/#0" 31
Anbochi, Atillo, Weigand
Taken from Rhys Alexeis reviewer. Digests from upperbatch.

raising the primary argument that a health care agreement is not an insurance
contract; hence the "incontestability clause" under the Insurance Code does not
apply.
Issue [1]: Whether a health care agreement between Philamcare and Ernani
Trinos is an insurance contract.
Held [1]: YES. Section 2 (1) of the Insurance Code defines a contract of
insurance as an agreement whereby one undertakes for a consideration to
indemnify another against loss, damage or liability arising from an unknown or
contingent event. An insurance contract exists where the following elements
concur: (1) The insured has an insurable interest; (2) The insured is subject to a
risk of loss by the happening of the designated peril; (3) The insurer assumes
the risk; (4) Such assumption of risk is part of a general scheme to distribute
actual losses among a large group of persons bearing a similar risk; and (5) In
consideration of the insurer's promise, the insured pays a premium. Section 3 of
the Insurance Code states that any contingent or unknown event, whether past
or future, which may damnify a person having an insurable interest against him,
may be insured against. Every person has an insurable interest in the life and
health of himself. Section 10 provides that "Every person has an insurable
interest in the life and health: (1) of himself, of his spouse and of his children;
(2) of any person on whom he depends wholly or in part for education or
support, or in whom he has a pecuniary interest; (3) of any person under a legal
obligation to him for the payment of money, respecting property or service, of
which death or illness might delay or prevent the performance; and (4) of any
person upon whose life any estate or interest vested in him depends." Herein,
the insurable interest of Trinos' husband in obtaining the health care agreement
was his own health. The health care agreement was in the nature of non-life
insurance, which is primarily a contract of indemnity. Once the member incurs
hospital, medical or any other expense arising from sickness, injury or other
stipulated contingent, the health care provider must pay for the same to the
extent agreed upon under the contract.
Issue [2]: Whether answers made in good faith, where matters of opinion or
judgment are called for, without intent to deceive will avoid a policy when they
were untrue.
Held [2]: NO. Where matters of opinion or judgment are called for, answers
made in good faith and without intent to deceive will not avoid a policy even
though they are untrue. Thus, although false, a representation of the
expectation, intention, belief, opinion, or judgment of the insured will not avoid
the policy if there is no actual fraud in inducing the acceptance of the risk, or its
acceptance at a lower rate of premium, and this is likewise the rule although the
statement is material to the risk, if the statement is obviously of the foregoing
character, since in such case the insurer is not justified in relying upon such
statement, but is obligated to make further inquiry. There is a clear distinction
between such a case and one in which the insured is fraudulently and
intentionally states to be true, as a matter of expectation or belief, that which he
then knows, to be actually untrue, or the impossibility of which is shown by the
facts within his knowledge, since in such case the intent to deceive the insurer
is obvious and amounts to actual fraud. The fraudulent intent on the part of the
insured must be established to warrant rescission of the insurance contract.
Concealment as a defense for the health care provider or insurer to avoid
liability is an affirmative defense and the duty to establish such defense by
satisfactory and convincing evidence rests upon the provider or insurer. In any
case, with or without the authority to investigate, Philamcare is liable for claims
made under the contract. Having assumed a responsibility under the
agreement, Philamcare is bound to answer the same to the extent agreed upon.
In the end, the liability of the health care provider attaches once the member is
hospitalized for the disease or injury covered by the agreement or whenever he
avails of the covered benefits which he has prepaid.
Issue [3]: Whether rescission must be exercised before commencement of an
action on the contract.
Held [3]: YES. Under Section 27 of the Insurance Code, "a concealment
entitles the injured party to rescind a contract of insurance." The right to rescind
should be exercised previous to the commencement of an action on the
contract. Herein, no rescission was made. Besides, the cancellation of health
care agreements as in insurance policies require the concurrence of the
following conditions: (1) Prior notice of cancellation to insured; (2) Notice must
be based on the occurrence after effective date of the policy of one or more of
the grounds mentioned; (3) Must be in writing, mailed or delivered to the insured
at the address shown in the policy; (4) Must state the grounds relied upon
provided in Section 64 of the Insurance Code and upon request of insured, to
furnish facts on which cancellation is based. None of the above pre-conditions
was fulfilled in this case. When the terms of insurance contract contain
limitations on liability, courts should construe them in such a way as to preclude
the insurer from non-compliance with his obligation. Being a contract of
adhesion, the terms of an insurance contract are to be construed strictly against
the party which prepared the contract the insurer. By reason of the exclusive
control of the insurance company over the terms and phraseology of the
insurance contract, ambiguity must be strictly interpreted against the insurer
and liberally in favor of the insured, especially to avoid forfeiture. This is equally
applicable to Health Care Agreements.

!"#$%&"'( * &++,- .$!/#0" 32
Anbochi, Atillo, Weigand
Taken from Rhys Alexeis reviewer. Digests from upperbatch.

Issue [4]: Whether the membership of the late Trinos is now incontestable.
Held [4]: YES. Under the title Claim procedures of expenses, Philamcare had
twelve months from the date of issuance of the Agreement within which to
contest the membership of the patient if he had previous ailment of asthma, and
six months from the issuance of the agreement if the patient was sick of
diabetes or hypertension. The periods having expired, the defense of
concealment or misrepresentation no longer lie.
Saura v Phil. International Co.
Facts: Saura Import & Export Co. Inc. Mortgages to the Phil. National Bank, a
parcel of land covered by a TCT in their name, to secure payment of a
promissory note in the amount of PhP27,000. On the said land was a building of
strong materials owned by the mortgagor Saura, which was always covered by
an insurance policy issued many years prior to the mortgage contract. Pursuant
to this, Saura insured the building and its contents against fire, for a period of
one year in the amount of PhP29,000, which policy was endorsed in favor of
PNB. The policy was delivered to the Bank by Saura, and barely 13 days after,
the insurer cancelled the insurance as of date of issue, with notice of
cancellation being sent by registered mail, to the bank in writing, personally
addresses to the Branch Manager of the Davao Branch. Later on, the building
was raged by fire, along with its contents. Notice of loss was presented by PNB
to Saura, and it was the first time the latter learned of the cancellation by the
insurer. Upon refusal of the Insurer to pay the amount of insurance, a case was
filed against the insurer, and later PNB was included as party defendant, after
its refusal to prosecute the case jointly with Saura.
Held: Fire Insurance policies and other contracts of insurance upon property, in
addition to the common provision for cancellation of the policy upon request of
the insured, generally provide for the cancellation by the insurer by notice to the
insured within a specified period, usually 5 days, and return of the unearned
portion of the premium paid. And such provision for cancellation upon notice
being authorized by statutes, either specifically or as a provision adopted in a
standard form of policy. The purpose of such is to prevent the cancellation of
the policy, without allowing the insured ample opportunity to negotiate for other
insurance in its stead. The form and sufficiency of the notice of cancellation is
determined by policy provisions. In order to form the basis for the cancellation of
the policy, notice need not be in a particular form, and it is sufficient, so long as
it positively and unequivocally indicates to the insured, that it is the intention of
the company that the policy shall cease to be binding. Where the policy
contains no provisions that a certain number of days notice shall be given, a
reasonable notice and opportunity to obtain other insurance must be given.
Actual personal notice to the insured is essential to the cancellation under a
provision for cancellation by notice. The actual condition precedent to a
cancellation of the policy by the insurer, and consequently, a letter containing
notice of cancellation which is mailed by the insurer but not received by the
insured, is ineffective as cancellation.
The policy here did not provide for the notice requirement, neither as to from or
period. The Insurance Law, does not likewise provide for such notice. This
being the case, it devolves upon the Court to apply the generally accepted
principles of insurance, regarding cancellation of the policy by the insurer. From
what has been stated, actual notice of cancellation in a clear and unequivocal
manner, preferably in writing must be made. In view of the importance of an
insurance contract, whould be given by the insurer to the insured, so that the
latter might be given and oppostunity to obtain other insurance policies for its
own protection. The notice should e personal to the insured, and/or through any
authorized person by the policy. In this case, the defendant company, must
have realized the paramount importance of sending a notice of cancellation,
when it sent the notice to the Bank, but not to the insured, which has direct
dealing with it. It was the primary duty of the company to notify the insured, at
the time when the policy was enforced and that under the facts, it is evident that
both the company and the bank failed, wittingly or unwittingly, to notify the
insured of the cancellation.

Sec. 67 76
Ang Giok Chip v Springfield Fire & Marine Insurance (56 Phil. 375; 1931)
Facts: Petitioner was an owner of a warehouse, situated atNo. 643 Calle Reina
Regente, City of Manila, containing goods insured with three insurance
company (totaling PhP60,000) where one of them was the Springfield, insurer
for the amount of PhP10,000.
In the policy, it is provided that the petitioner will not store more than three
percent of fire hazardous materials on the warehouse.
The warehouse was burn and so the petitioner filed for a claim of the insurance
policy but the Springfield refused claiming that the petitioner violated the
contents of the insurance policy.
Issue: WHETHER OR NOT, THE INSURANCE POLICY BETWEEN THE
PARTY IN THIS CASE IS VALID.

!"#$%&"'( * &++,- .$!/#0" 33
Anbochi, Atillo, Weigand
Taken from Rhys Alexeis reviewer. Digests from upperbatch.

Held: According to the Supreme Court, Act No. 2427, as amended, section 65
reading: "Every express warranty, made at or before the execution of a policy,
must be contained in the policy itself, or in another instrument signed by the
insured and referred to in the policy, as making a part of it." And since the party
freely entered in to such policy, it is therefore binding between them. More than
3 per cent of the total value of the merchandise contained in the warehouse
constituted hazardous goods, and that this per cent reached as high as 39.
Reliance was placed on the consular invoices and on the testimony of the
adjuster, Herridge. Section 65 of the Philippine Insurance Act corresponds to
section 2605 of the Civil Cod of California. The comments of the Code
Examiners of California disclose that the language of section 2605 was quite
different from that under the Code as adopted in 1872. That language was
found too harsh as to insurance companies. The Code Examiners' notes state:
"The amendment restores the law as it existed previous to the Code: See
Parsons on Maritime Law, 106, and Phillips on Insurance, sec. 756." The
passage referred to in Philips on Insurance, was worded by the author as
follows:
"Any express warranty or condition is always a part of the policy, but, like any
other part of an express contract, may be written in the margin, or containedin
proposals or documents expressly referred to in the policy, and so made a part
of it." The annotator of the Civil Code of California, after setting forth these facts,
adds:
. . . The section as it now reads is in harmony with the rule that a warranty may
be contained in another instrument than the policy when expressly referredto in
the policy as forming a part thereof: . . . .
We, therefore, think it wrong to hold that the California law represents a radical
departure from the basic principles governing the law of insurance. We are
more inclined to believe that the codification of the law of California had exactly
the opposite purpose, and that in the language of the Federal courtit was but an
affirmance of the generally accepted doctrine applicable to such contracts. This
being true, we turn to two of such well recognized doctrines.
In the first place, it is well settled that a rider attached to a policy is a part of the
contract, to the same extent and with like effect as it actually embodied therein.
(I Couch, Cyclopedia of Insurance Law, sec. 159.) In the second place, it is
equally well settled that an express warranty must appearupon the face of the
policy, or be clearly incorporated therein and made a part thereof by explicit
reference, or by words clearly evidencing such intention. (4 Couch, Cyclopedia
of Insurance Law, sec. 862.) The law says that every express warranty must be
"contained in the policy itself." The word "contained," according to the
dictionaries, means "included," inclosed," "embraced," "comprehended," etc.
When, therefore, the courts speak of a rider attached to the policy, and thus
"embodied" therein, or of a warranty "incorporated" in the policy, it is believed
that the phrase "contained in the policy itself" must necessarily include such
rider and warranty. As to the alternative relating to "another instrument,"
"instrument" as here used could not mean a mere slip of paper like a rider, but
something akin to the policy itself, which in section 48 of the Insurance Act is
defined as "The written instrument, in which a contract of insurance is set forth."
In California, every paper writing is not necessarily an "instrument" within the
statutory meaning of the term. The word "instrument has a well defined
definition in California, and as used in the Codes invariably means some written
paper or instrument signed and delivered by one person to another, transferring
the title to, or giving a lien, on property, or giving a right to debt or duty. (Hoag
vs. Howard [1880], 55 Cal., 564; People vs. Fraser[1913], 137 Pac., 276.)
In other words, the rider, stating the prohibition against storing more than 3%
haradous materails, is contained in the policy itself, because by the contract of
insurance agreed to by the parties it is made to form a part of the same, but is
not another instrument signed by the insured and referred to in the policy as
forming a part of it. Again, referring to the jurisprudence of California, another
rule of insurance adopted in that State is in point. It is admitted that the policy
before us was accepted by the plaintiff. The receipt of this policy by the insured
without objection binds both the acceptor and the insured to the terms thereof.
The insured may not thereafter be heard to say that he did not read the policy or
know its terms, since it is his duty to read his policy and it will be assumed that
he did so. In California Jurisprudence, vol. 14, p. 427, from which these
statements are taken with citations to California decisions, it is added that it has
been held that where the holder of a policy discovers a mistake made by
himself and the local agent in attaching the wrong rider to his application, elects
to retain the policy issued to him, and neither requests the issuance of a
different one nor offers to pay the premium requisite to insure against the risk
which he believe the rider to cover, he thereby accepts the policy.

We are given to understand, and there is no indication to the contrary, that we
have here a standard insurance policy. We are further given to understand, and
there is no indication to the contrary, that the issuance of the policy in this case
with its attached rider conforms to well established practice in the Philippines
and elsewhere. We are further given to understand, and there is no indication to
the contrary, that there are no less than sixty-nine insurance companies doing
business in the Philippine Islands with outstanding policies more or less similar

!"#$%&"'( * &++,- .$!/#0" 34
Anbochi, Atillo, Weigand
Taken from Rhys Alexeis reviewer. Digests from upperbatch.

to the one involved in this case, and that to nullify such policies would place an
unnecessary hindrance in the transaction of insurance business in the
Philippines. These are matters of public policy. We cannot believe that it was
ever the legislative intention to insert in the Philippine Law on Insurance an
oddity, an incongruity, entirely out of harmony with the law as found in other
jurisdiction, and destructive of good business practice. We have studied this
case carefully and having done so have reached the definite conclusion that
warranty F, a rider attached to the face of the insurance policy, and referred to
in contract of insurance, is valid and sufficient under section 65 of the Insurance
Act.
Young v Midland Textile Ins. (30 Phil. 617; 1915)
Facts: The plaintiff conducted a candy and fruit store on the Escolta, in the city
of Manila, and occupied a building at 321 Calle Claveria, as a residence and
bodega (storehouse).
On the 29th of May, 1912, the defendant, in consideration of the payment of a
premium of P60, entered into a contract of insurance with the plaintiff (policy
No. 509105) by the terms of which the defendant company, upon certain
conditions, promised to pay to the plaintiff the sum of P3,000, in case said
residence and bodega and contends should be destroyed by fire.
On the conditions of said contract of insurance is found in "warranty B" and is
as follows: "Waranty B. It is hereby declared and agreed that during the
pendency of this policy no hazardous goods stored or kept for sale, and no
hazardous trade or process be carried on, in the building to which this insurance
applies, or in any building connected therewith."
On the 4th or 5th of February, 1913, the plaintiff placed in said residence and
bodega three boxes, 18 by 18 by 20 inches measurement, which belonged to
him and which were filed with fireworks.
On the 18th day of March, 1913, said residence and bodega and the contents
thereof were partially destroyed by fire. Said fireworks had been given to the
plaintiff by the former owner of the Luneta Candy Store; that the plaintiff
intended to use the same in the celebration of the Chinese new year; that the
authorities of the city of Manila had prohibited the use of fireworks on said
occasion, and that the plaintiff then placed the same in said bodega, where they
remained from the 4th or 5th of February, 1913, until after the fire of the 18th of
March, 1913. Both of the parties agree that said fireworks come within the
phrase "hazardous goods," mentioned in said "warranty B" of the policy. That
said fireworks were found in a part of the building not destroyed by the fire; that
they in no way contributed to the fire, or to the loss occasioned thereby.
Issue: Whether or not the placing of said fireworks in the building insured,
under the conditions above enumerated, they being "hazardous goods," is a
violation of the terms of the contract of insurance and especially of "warranty B."
Held: "Warranty B" provides that "no hazardous goods be stored" in the building
insured. It is admitted by both parties that the fireworks are "hazardous goods."
The defendant alleged that they were "stored." The plaintiff contends that under
all the facts and circumstances of the case, they were not "stored" in said
building, and that the placing of them in the building was not a violation of the
terms of the contract. Both the plaintiff and defendant agree that if they were
"hazardous goods," and if they were "stored," then the act of the plaintiff was a
violation of the terms of the contract of insurance and the defendant was
justified in repudiating its liability thereunder. This leads us to a consideration of
the meaning of the accord "stored" as used in said "warranty B." While the word
"stored" has been variously defined by authors, as well as by courts, we have
found no case exactly analogous to the present. The plaintiff says that he
placed said fireworks in the bodega after he had been notified that he could not
use them on the Chinese new year, in order that he might later send them to a
friend in the provinces. Whether a particular article is "stored" or not must, in
some degree, depend upon the intention of the parties. The interpretation of the
word "stored" is quite difficult, in view of the many decisions upon the various
conditions presented. Nearly all of the cases cited by the lower court are cases
where the article was being put to some reasonable and actual use, which
might easily have been permitted by the terms of the policy, and within the
intention of the parties, and excepted from the operation of the warranty, like the
present. Said decision are upon cases like:
1. Where merchants have had or kept the "hazardous" articles in small
quantities, and for actual daily use, for safe, such as gasoline, gunpowder, etc.;
2. Where such articles have been brought on the premises for actual use
thereon, and in small quantities, such as oil, paints, etc; and
3. Where such articles or goods were used for lighting purpose, and in small
quantities.
The author of the Century Dictionary defines the world "store" to be a deposit in
a store or warehouse for preservation or safe keeping; o place in a warehouse
or other place of deposit for safe keeping. See also the definitions given by the
Standard Dictionary, to the same effect.

!"#$%&"'( * &++,- .$!/#0" 35
Anbochi, Atillo, Weigand
Taken from Rhys Alexeis reviewer. Digests from upperbatch.

Said definitions, of course, do not include a deposit in a store, in small
quantities, for daily use. "Daily use" precludes the idea of a deposit for
preservation or safe keeping, as well as a deposit for future consumption, or
safe keeping.
In the present case no claim is made that the "hazardous goods" were placed in
the bodega for present or daily use. It is admitted that they were placed in the
bodega "for future use," or for future consumption, or for safe keeping. The
plaintiff makes no claim that he deposited them there with any other idea than
"for future use" for future consumption. It seems clear to us that the
"hazardous goods" in question were "stored" in the bodega, as that word is
generally defined. That being true, suppose the defendant had made an
examination of the premises, even in the absence of a fire, and had found he
"hazardous goods" there, under the conditions above described, would it not
have been justified, then and there, in declaring the policy null and of no effect
by reason of a violation of its terms on he par of the plaintiff? If it might, then
may it no repudiate is liability, even after the fire? If the "warranty" is a term of
the contract, will not its violation cause a breach and justify noncompliance or a
repudiation?
Contracts of insurance are contracts of indemnity upon the terms and conditions
specified in the policy. The parties have a right to impose such reasonable
conditions at the time of the making of the contract as they may deem wise and
necessary. The rate of premium is measured by the character of the risk
assumed. The insurance company, for a comparatively small consideration,
undertakes to guarantee the insured against loss or damage, upon the terms
and conditions agreed upon, and upon no other, and when called upon to pay,
in case of loss, the insurer, therefore, may justly insist upon a fulfillment of these
terms. If the insured cannot bring himself within the conditions of the policy, he
is not entitled to recover for the loss. The terms of the policy constitute the
measure of the insurer's liability, and in order to recover the insured must show
himself within those terms; and if it appears that the contract has been
terminated by a violation, on the part of the insured, of its conditions, then there
can be no right of recovery. The compliance of the insured with the terms of the
contract is a condition precedent to the right of recovery. If the insured has
violated or failed to perform the conditions of the contract, and such a violation
or want of performance has not been waived by the insurer, then the insured
cannot recover. Courts are not permitted to make contracts for the parties. The
function and duty of the courts consist simply in enforcing and carrying out he
contracts actually made. While it is true, as a general rule, that contracts of
insurance are construed most favorably to the insured, yet contracts of
insurance, like other contracts, are to be construed according to the sense and
meaning of the terms which the parties themselves have used. If such terms are
clear and unambiguous they must be taken and understood in their plain,
ordinary and popular sense. The conditions of contracts of insurance, when
plainly expressed in a policy, are binding upon the parties and should be
enforced by the courts, if the evidence brings the case clearly within their
meaning and intent. It tends to bring the law itself into disrepute when, by astute
and subtle distinctions, a plain case is attempted to be taken without the
operation of a clear, reasonable, and material obligation of the contract.
An increase of risk which is substantial and which is continued for a
considerable period of time, is a direct and certain injury to the insurer, and
changes the basis upon which the contract of insurance rests. Therefore and for
the foregoing reasons, the judgment of the lower court is hereby revoked and
the defendant is hereby relieved from any responsibility under said complaint,
and, without any finding as to costs, it is so ordered.
Gen. Insurance v Ng Hua (106 Phil 1117; 1960)
Facts: On April 15, 1952, the defendant General Insurance and Surety
Corporation issued its insurance Policy No. 471, insuring against fire, for one
year, the stock in trade of the Central Pomade Factory owned by Ng Hua, the
court insured. The next day, the Pomade factory building burned, resulting in
destruction by fire of the insured properties. Ng Hua claimed indemnity from the
insurer. The policy covered damages up to P10,000.00; but after some
negotiations and upon suggestion of the Manila Adjustment Company, he
reduced the claim of P5,000.00. Nevertheless, the defendant insurer refused to
pay for various reasons, namely (a) action was not filed in time; (b) violation of
warranty; (c) submission of fraudulent claim; and (f) failure to pay the premium.
The aforesaid Policy No. 471 contains this stipulation on the back thereof;.
3. The insured shall give notice to the company of any insurance or insurances
already affected, or which may subsequently be effected, covering any of the
property hereby insured, and unless such notice be given and the particulars of
such insurance or insurances be stated in or endorsed on this Policy by or on
behalf of the Company before the occurrence of any loss or damage, all
benefits under the policy shall be forfeited.
The face of the policy bore the annotation: "Co-Insurance Declared NIL". It is
undenied that Ng Hua had obtained fire insurance on the same goods, for the
same period of time, in the amount of P20,000.00 from General Indemnity Co.
However, the Court of Appeals referring to the annotation and overruling the
defense, held that there was no violation of the above clause, inasmuch as "co-
insurance exists when a condition of the policy requires the insured to bear

!"#$%&"'( * &++,- .$!/#0" 36
Anbochi, Atillo, Weigand
Taken from Rhys Alexeis reviewer. Digests from upperbatch.

ratable proportion of the loss when the value of the insured property exceeds
the face value of the policy," hence there is no co-insurance here.
Issue: Whether or not there was warranty breach or concealment of the other
insurance and/or violation of the provision of the policy above-mentioned.
Held: Yes. Undoubtedly, co-insurance exists under the condition described by
the appellate court. But that is one kind of co-insurance. It is not the only
situation where co-insurance exists. Other insurers of the same property against
the same hazard are sometimes referred as co-insurers and the ensuing
combination as co-insurance.1 And considering the terms of the policy which
required the insured to declare other insurances, the statement in question must
be deemed to be a statement (warranty) binding on both insurer and insured,
that there were no other insurance on the property. Remember it runs "Co-
Insurance declared"; emphasis on the last word. If "Co-Insurance" means that
the Court of Appeals says, the annotation served no purpose. It would even be
contrary to the policy itself, which in its clause No. 17 made the insured a co-
insurer for the excess of the value of the property over the amount of the policy.
The annotation then, must be deemed to be a warranty that the property was
not insured by any other policy. Violation thereof entitles the insurer to rescind.
(Sec. 69. Insurance Act) Such misrepresentation is fatal in the light of our views
in Santa Ana vs. Commercial Union Assurance Company, Ltd., 55 Phil., 329.
The materiality of non-disclosure of other insurance policies is not open to
doubt.
Furthermore, even if the annotations were overlooked, the defendant insurer
would still be free from liability because there is no question that the policy
issued by General Indemnity had not been stated in nor endorsed on Policy No.
471 of defendant. And as stipulated in the above-quoted provisions of such
policy "all benefit under this policy shall be forfeited."
To avoid the dissastrous effect of the misrepresentation or concealment of the
other insurance policy, Ng Hua alleges "actual knowledge" on the part of
General insurance of the fact that he had taken out additional insurance with
General Indemnity. He does not say when such knowledge was acquired or
imparted. If General Insurance know before issuing its policy or before the fire,
such knowledge might overcome the insurer's defense. However, the Court of
Appeals found no evidence of such knowledge. We have read the pages of the
stenographic notes cited by Ng Hua and we all gather is evidence of the
existence of the Insurance General Indemnity Company. As to knowledge of
General Insurance before issuance of its policy or the fire, there was none.
Indeed, this concealment and violation was expressly set up as a special
defense in the answer. Yet plaintiff did not, in avoidance, reply nor assert such
knowledge. And it is doubtful whether the evidence on the point would be
admissible under the pleadings.
Sections 77-81
Philippine Phoenix v Woodworks (1967)
Facts: On April 1, 1960, Phoenix issued to Woodworks a Fire Policy for the
amount of PhP300,000. The premiums on the policy amounted to Php6,051.95,
and the margin fee amounted to PhP 363.32, as per the adopted plan as per
implementation of RA 2609. Woodworks was able to pay PhP3,000 on Sept. 22,
1960.Several demands were made by Phoenix for Woodworks to pay the
remaining premiums, which amounted to Php3,522.09. For Woodworks failure
to pay, Phoenix instituted an action to recover the unpaid premiums.
Woodworks contended that the non-payment of the balance of the premiums
resulted in the cancellation of the contract of insurance, and thus Phoenix no
longer had a cause of action against them.
Issue: W/N the non-payment of the balance of the premium resulted in the
cancellation of the contract of insurance.
Held: No. It is to be noted that Woodworks already paid PhP3,000 of the said
premiums. There is, consequently, no doubt that as between the insurer and the
insured, there was already a perfected contract of insurance but a partially
performed one as far as the payment of the premium was concerned.
Thereafter, the obligation of the insurer to pay the insured the amount for which
the policy was issued in case the conditions therefore had been complied with,
arose and became binding upon it while the obligation of the insured to pay the
remainder of the total amount of the premium due became demandable. The
Court cannot agree with the appellants theory that non-payment of the premium
due produced the cancellation of the contract of insurance. Such theory would
place exclusively in the hands of the insured the right to decide whether the
contract should stand or not. The correct view, would seem that as the contract
had become perfected, the parties could demand from each other the
performance of whatever obligations they had assumed. In the case of the
insurer, it is obvious that it had the right to demand from the insured the
completion of the payment of the premium due or sue for the rescission of the
contract. As it chose to demand specific performance of the insureds obligation
to pay the balance of the premium, the latters duty to pay is indeed indubitable.


!"#$%&"'( * &++,- .$!/#0" 37
Anbochi, Atillo, Weigand
Taken from Rhys Alexeis reviewer. Digests from upperbatch.

Philippine Phoenix v Woodworks (1979)
Facts: On July 21, 1960, Phoenix issued to Woodworks a fire insurance policy
for Php 500,000 for the lattersbuilding machinery and equipment for a term of 1
year from issue date until July 21, 1961. Premiums and other charges including
the margin fee imposed by law amounted to PhP590.36 and the DST in the
amount of PhP 156.60 on the policy, amounted to PhP 10,593.36. Again, as in
the previous case, Woodworks failed to pay, either upon issuance of the policy
or anytime thereafter. Before the expiration of the 1 year term, Phoenix notified
Woodworks of the cancellation of the fire insurance policy allegedly upon the
latters request. Woodworks denied making such a request. Phoenix credited
woodworks with the amount of PhP 3,110.25 for the unexpired period of 96
days and claimed payment for the balance of Php7,483.11 representing earned
premium from issue date until April 1961, for a total of 271 days. Woodworks
refused to pay the earned premium because the insurer did not stand liable for
any indemnity during the period that the premium was not paid. Phoenix thus
instituted the instant case against Woodworks for the payment of the earned
premiums.
Issue: W/N Woodworks is liable for the earned premiums for 271 days before
the contract was said to have been cancelled.
Held: No. Insurance is "a contract whereby one undertakes for a consideration
to indemnify another against loss, damage or liability arising from an unknown
or contingent event." 5 The consideration is the "premium". "The premium must
be paid at the time and in the way and manner specified in the policy and, if not
so paid, the policy will lapse and be forfeited by its own terms." 6 chanrobles
virtual law library
Paragraph "2" of the Policy further contained the following condition:

2. No payment in respect of any premium shall be deemed to be payment
to the Company unless a printed form of receipt for the same signed by an
Official or duly-appointed Agent of the Company shall have been given to
the Insured.

Paragraph "10" of the Policy also provided:

10. This insurance may be terminated at any time at the request of the Insured,
in which case the Company will retain the customary short period rate for the
time the policy has been in force. This insurance may also at any time be
terminated at the option of the Company, on notice to that effect being given to
the Insured, in which case the Company shall be liable to repay on demand a
ratable proportion of the premium for the unexpired term from the date of the
cancelment.

Clearly, the Policy provides for pre-payment of premium. Accordingly; "when the
policy is tendered the insured must pay the premium unless credit is given or
there is a waiver, or some agreement obviating the necessity for prepayment."
To constitute an extension of credit there must be a clear and express
agreement therefor." From the Policy provisions, we fail to find any clear
agreement that a credit extension was accorded defendant. And even if it were
to be presumed that plaintiff had extended credit from the circumstances of the
unconditional delivery of the Policy without prepayment of the premium, yet it is
obvious that defendant had not accepted the insurer's offer to extend credit,
which is essential for the validity of such agreement.
Since the premium had not been paid, the policy must be deemed to have
lapsed. The non-payment of premiums does not merely suspend but put, an
end to an insurance contract, since the time of the payment is peculiarly of the
essence of the contract. ... the rule is that under policy provisions that upon the
failure to make a payment of a premium or assessment at the time provided for,
the policy shall become void or forfeited, or the obligation of the insurer shall
cease, or words to like effect, because the contract so prescribes and because
such a stipulation is a material and essential part of the contract. This is true, for
instance, in the case of life, health and accident, fire and hail insurance policies.
In fact, if the peril insured against had occurred, plaintiff, as insurer, would have
had a valid defense against recovery under the Policy it had issued. Explicit in
the Policy itself is plaintiff's agreement to indemnify defendant for loss by fire
only "after payment of premium," supra. Compliance by the insured with the
terms of the contract is a condition precedent to the right of recovery. The
burden is on an insured to keep a policy in force by the payment of premiums,
rather than on the insurer to exert every effort to prevent the insured from
allowing a policy to elapse through a failure to make premium payments. The
continuance of the insurer's obligation is conditional upon the payment of
premiums, so that no recovery can be had upon a lapsed policy, the contractual
relation between the parties having ceased.


!"#$%&"'( * &++,- .$!/#0" 38
Anbochi, Atillo, Weigand
Taken from Rhys Alexeis reviewer. Digests from upperbatch.

Moreover, "an insurer cannot treat a contract as valid for the purpose of
collecting premiums and invalid for the purpose of indemnity."
N.B. This case differs from the previous one, in one important aspect, in the
previous one, there was already partial payment of the premiums, but here, no
premiums whatsoever was paid, thus, no policy, and no right to demand
payment on the part of the insurer.
Capital Insurance v Plastic Era
Facts: Capital Insurance issued a fire insurance policy in favor of Plastic Era,
covering the latters building, equipments, raw materials, products and
accessories located at Sheridan Street, Mandaluyong, Rizal. The policy
expressly provides that if the property insured would be destroyed or damaged
by fire after the payment of the premiums, at anytime between the 15th day of
December 1960 and one o'clock in the afternoon of the 15th day of December
1961, the insurance company shall make good all such loss or damage in an
amount not exceeding P100,000.00. When the policy was delivered, Plastic Era
failed to pay the corresponding insurance premium. However, through its duly
authorized representative, it issued an acknowledgement receipt wherein it
promised to pay PhP 2,200.00, 30 days after effective date. Thereafter, partial
payment of PhP 1,000 was made, through a postdated check, but when the
insurer deposited the same, it was dishonored for lack of funds. 2 days after the
premium became due, the property of Plastic Era was destroyed by fire, for
which it filed its claim with Capital Insurance. In less than a month Plastic Era
demanded from Capital Insurance the payment of the sum of P100,000.00 as
indemnity for the loss of the insured property under the fire insurance policy, but
the latter refused for the reason that, among others, Plastic Era failed to pay the
insurance premium. The instant case was filed by Plastic Era to recover the
amount of PhP100,000.
Issue: W/N a contract of insurance has been duly perfected between the
petitioner, Capital Insurance, and respondent Plastic Era.
Held: Yes. In clear and unequivocal terms the insurance policy provides that it
is only upon payment of the premiums by Plastic Era that Capital Insurance
agrees to insure the properties of the former against loss or damage in an
amount not exceeding P100,000.00. The crux of the problem then is whether at
the time the insurance policy was delivered to Plastic Era on December 17,
1960, the latter was able to pay the stipulated premium. It appears on record
that on the day the insurance policy was delivered, Plastic Era did not pay the
Capital Insurance, but instead executed an acknowledgment receipt of Policy
No. 22760. In said receipt Plastic Era promised to pay the premium within thirty
(30) days from the effectivity date of the policy on December 17, 1960 and
Capital Insurance accepted it. What then is the effect of accepting such
acknowledgment receipt from the Plastic Era? Did the Capital Insurance mean
to agree to make good its undertaking under the policy if the premium could be
paid on or before January 16, 1961? And what would be the effect of the
delivery to Capital Insurance on January 8, 1961 of a postdated check (January
16, 1961) in the amount of P1,000.00, payable to the order of the latter? Could
not this have been considered a valid payment of the insurance premium?
Under Art. 1249 of the Civil Code, the mere delivery of a bill of exchange in
payment of a debt does not immediately effect payment. It simply suspends the
action arising from the original obligation in satisfaction of which it was
delivered, until payment is accomplished either actually or presumptively.
Tender of draft or check in order to effect payment that would extinguish the
debtor's liability should be actually cashed. If the delivery of the check of Plastic
Era to Capital Insurance were to be viewed in the light of the foregoing, no
payment of the premium had been effected, for it is only when the check is
cashed that it is said to effect payment. Significantly, in this case, Capital
Insurance accepted the promise of Plastic Era to pay the insurance premium
within thirty (30) days from the effective date of policy. By so doing, it has
implicitly agreed to modify the tenor of the insurance policy and in effect, waived
the provision therein that it would only pay for the loss or damage in case the
same occurs after the payment of the premium. Considering that the
insurance policy is silent as to the mode of payment, Capital Insurance is
deemed to have accepted the promissory note in payment of the premium.
This rendered the policy immediately operative on the date it was delivered.
Capital Insurance accepted the acknowledgment receipt of the Plastic Era
promising to pay the insurance premium within thirty (30) days from December
17, 1960. Hence, when the damage or loss of the insured property occurred,
the insurance policy was in full force and effect. The fact that the check issued
by Plastic Era in partial payment of the promissory note was later on dishonored
did not in any way operate as a forfeiture of its rights under the policy, there
being no express stipulation therein to that effect. Capital Insurance had
accepted a check for P1,000.00 from Plastic Era in partial payment of the
premium on the insurance policy. Although the check was due for payment on
January 16, 1961 and Plastic Era had sufficient funds to cover it as of January
19, 1961, Capital Insurance decided to hold the same for thirty-five (35) days
before presenting it for payment. Having held the check for such an
unreasonable period of time, Capital Insurance was estopped from claiming a
forfeiture of its policy for non-payment even if the check had been dishonored
later. Thus, Plastic Era has complied with its obligation to pay the insurance
premium and therefore Capital Insurance is obliged to make good its
undertaking to Plastic Era.

!"#$%&"'( * &++,- .$!/#0" 39
Anbochi, Atillo, Weigand
Taken from Rhys Alexeis reviewer. Digests from upperbatch.

Valenzuela v CA
Facts: Arturo Valenzuela was a General Agent of the Phil. American General
Insurance Company, since 1965. He was authorized to solicit and sell in behalf
of Philamgen all kinds of non-life insurance, and in consideration of his services,
he was entitiled to receive the full agents commission of 32.5% from
Philamgen. From 1973 to 1975, he solicited marine insurance form Delta
Motors Inc. in the amount of P4.4 Million from which he was entitled to a
commission of 32%. However, he did not receive his full commission which
amounted to P1.6 Million from the P4.4 Million insurance coverage of the Delta
Motors. During the period 1976 to 1978, premium payments amounting to
P1,946,886.00 were paid directly to Philamgen and Valenzuela's commission to
which he is entitled amounted to P632,737.00. In 1977, Philamgen started to be
interested in and expressed its intent to share in the commission due to
Valenzuela on a 50-50 basis, but Valenzuela refused. In 1978, Philamgen and
its president (Aragon), insisted on the sharing of the commission with
Valenzuela, followed up by another sharing proposal dated June 1, 1978. On
June 16, 1978, Valenzuela firmly reiterated his objection in sharing his
commissions with Philamgen. He stated that: "It is with great reluctance that I
have to decline upon request to signify my conformity to your alternative
proposal regarding the payment of the commission due me. However, I have no
choice for to do otherwise would be violative of the Agency Agreement
executed between our goodselves." Because of his refusal, Philamgen reversed
the commission due to him, by not crediting in his account the commissions
earned from Delta Motors, placed the agency transactions on a cash and carry
basis, and started to leak out new that Valenzuela has a substantial account
with Philamgen. Valenzuelas business as an insurance agent declined, and
later on, Philamgen terminated the general agency agreement with Valenzuela
based on such grounds. Delta Motors, however, in the meantime, failed to pay
the premiums in the amount of PhP 1,900,000.00. A suit for damages was filed
by Valenzuela due to loss of business and his share in the commission form
Delta Motors.
Issue: W/N Valenzuela is liable for the unpaid premiums of Delta Motors, as
held by the CA.
Held: No. In the insurance business in the Philippines, the most difficult and
frustrating period is the solicitation and persuasion of the prospective clients to
buy insurance policies. Normally, agents would encounter much
embarrassment, difficulties, and oftentimes frustrations in the solicitation and
procurement of the insurance policies. To sell policies, an agent exerts great
effort, patience, perseverance, ingenuity, tact, imagination, time and money. In
the case of Valenzuela, he was able to build up an Agency from scratch in 1965
to a highly productive enterprise with gross billings of about Two Million Five
Hundred Thousand Pesos (P2,500,000.00) premiums per annum. The records
sustain the finding that the private respondent started to covet a share of the
insurance business that Valenzuela had built up, developed and nurtured to
profitability through over thirteen (13) years of patient work and perseverance.
When Valenzuela refused to share his commission in the Delta account, the
boom suddenly fell on him.
The private respondents by the simple expedient of terminating the General
Agency Agreement appropriated the entire insurance business of Valenzuela.
With the termination of the General Agency Agreement, Valenzuela would no
longer be entitled to commission on the renewal of insurance policies of clients
sourced from his agency. Worse, despite the termination of the agency,
Philamgen continued to hold Valenzuela jointly and severally liable with the
insured for unpaid premiums.
As to the issue of whether or not the petitioners are liable to Philamgen for the
unpaid and uncollected premiums which the respondent court ordered
Valenzuela to pay Philamgen the amount of One Million Nine Hundred Thirty-
Two Thousand Five Hundred Thirty-Two and 17/100 Pesos (P1,932,532,17)
with legal interest thereon until fully paid, the Court ruled that the respondent
court erred in holding Valenzuela liable. Under Section 77 of the Insurance
Code, the remedy for the non-payment of the insurance Premium is to put an
end to and render the insurance policy not binding. In Philippine Phoenix Surety
and Insurance, Inc. v. Woodworks, Inc. [1979] the Court held that the non-
payment of premium does not merely suspend but puts an end to an insurance
contract since the time of the payment is peculiarly of the essence of the
contract. And in Arce v. The Capital Insurance and Surety Co. Inc. we the rule
was reiterated that unless premium is paid, an insurance contract does not take
effect. Perforce, since admittedly the premiums have not been paid, the policies
issued have lapsed. The insurance coverage did not go into effect or did not
continue and the obligation of Philamgen as insurer ceased. Hence, for
Philamgen which had no more liability under the lapsed and inexistent
policies to demand, much less sue Valenzuela for the unpaid premiums
would be the height of injustice and unfair dealing. In this instance, with
the lapsing of the policies through the nonpayment of premiums by the
insured there were no more insurance contracts to speak of. As the Court
held in the Philippine Phoenix Surety case, supra "the non-payment of
premiums does not merely suspend but puts an end to an insurance contract
since the time of the payment is peculiarly of the essence of the contract."
Areola v CA

!"#$%&"'( * &++,- .$!/#0" 4:
Anbochi, Atillo, Weigand
Taken from Rhys Alexeis reviewer. Digests from upperbatch.

Facts: Areola applied for and was issued a one year personal accident
insurance policy with Prudential Assurance. He made an initial payment for
which a collectors provisional receipt was issued (because it was only
Prudentials agent who collected the initial premium). The provisional receipt
stated that the Official Receipt must be sent within 7 days from the issuance of
the provisional receipt and should Areola not receiving it on time, he should
inform the main office immediately. Areola never received the Official Receipt
and his policy was thereafter cancelled for non-payment of premiums. When
Areola brought the matter to the head office, it was discovered that the agent
who collected the premium failed to remit the same to the head office.
Apologizing for the inconvenience, Prudential agreed to re-instate the policy and
even extend it lifetime. Before this could happen, however, a complaint for
damages arising from breach of contract was already filed by Santos against
Prudential Prudential contends that Santos had no cause of action as the error
had been timely rectified and that Areola never lost insurance coverage it
appearing that Prudential agreed to make the policy enforceable until all things
had been cleared up.
Issue: W/N Prudential is liable for the unilateral cancellation of the policy.
Held: Yes. The act of the agent is considered the act of Prudential. It is of no
moment that Prudential was also defrauded by the agent because of the non-
accrual of the payment. In the law of agency, the agent who acts within the
scope of his authority binds his principal. The fact that Prudential immediately
rectified the error is not a bar for the recovery of damages. In the law of
contracts, an aggrieved party if given the choice of either rescission or specific
performance. In either case, the aggrieved party may claim damages as well
from the defaulting party. However, in this case only nominal damages in the
amount of 30,000 can be awarded. Nominal damages are "recoverable where a
legal right is technically violated and must be vindicated against an invasion that
has produced no actual present loss of any kind, or where there has been a
breach of contract and no substantial injury or actual damages whatsoever have
been or can be shown.
Tibay v CA
Facts: Insurer Fortune Life issued fire insurance policy in favor of Tibay and/or
Roraldo on their 2-storey residential building, with all their personal effects
therein for 600K covering January 1987 1988. Of the total premium PhP
2,900, Tibay only paid PhP 600.

March 1987, the building was completely burned. 2 days after the incident,
Tibay paid the balance on the premium and on the same day filed a claim with
Fortune. The latter denied the claim for violating a condition in the policy and
under Sec 77. Tibay then sued to recover PhP 600,000. The Trial court ruled
for Tibay. CA reversed, but ordered Fortune to return the premium.
Issue: W/N insurance proceeds may be collected on a partially paid premium.
Held: No. Insurance is a contract whereby one undertakes for a consideration
to indemnify another against loss blah blah. That consideration is the premium.
Under the policy, the condition requires that it shall not be in force until the
premium has been fully paid and any agreement seeking to amend this
condition shall be invalid. Except in cases where rules may hereafter provide for
payment of the premiums in periodic instalments, it is hereby declared that this
policy shall be deemed valid and binding only when the premium have been
paid in full and duly accepted and receipted.
Petitioners insist that Fortune is liable despite partial payment of the premium,
citing the first Philippine Phoenix case and Makati Tuscany case. In Phoenix,
by accepting the initial payment of PhP3,000 and then demanding the
remainder of the premium without any other precondition to its enforceability,
the insurer had shown its intention to continue with the existing contract of
insurance. This case is different, because the parties expressly agreed that no
policy shall take effect until full payment was given prior to the occurrence of the
risk insured against. In Makati, parties mutually agreed as regards the payment
in installments. These cases demonstrate a waiver of prepayment in full by the
insurer, unlike in the present case. The instalments herein made do not
contemplate payment, but rather a deposit to be held in trust by the insurer until
the full amount has been tendered. It is elemental law that payment of premium
is requisite to keep the policy of insurance in force. If the premium is not paid in
the manner prescribed in the policy as intended by the parties the policy is
ineffective. Partial payment even when accepted as a partial payment will not
keep the policy alive even for such fractional part of the year as the part
payment bears to the whole payment. The case of South Sea Surety (in the
Syllabus but not assigned) speaks only of 2 exceptions to the payment of the
entire premium as a prerequisite to the validity of the insurance contract: (1) in
case the insurance coverage relates to life or industrial life (health) insurance
when a grace period applies, and (2) when the insurer makes a written
acknowledgment of the receipt of premium, this acknowledgment being
declared by law to, be then conclusive evidence of the premium payment.


!"#$%&"'( * &++,- .$!/#0" 41
Anbochi, Atillo, Weigand
Taken from Rhys Alexeis reviewer. Digests from upperbatch.

In the absence of clear waiver of prepayment in full by the insurer, the insured
cannot collect on the proceeds of the policy. For as long as the current
Insurance Code remains unchanged and partial payment of premiums is
not mentioned at all as among the exceptions provided in Secs. 77 and 78,
no policy of insurance can ever pretend to be efficacious or effective until
premium has been fully paid.
UCPB v Masagana Telemart (2001 MR)
Facts: Masagana obtained 5insurance policies from UCPB. The effectivity form
of the policies as reflected on its face is 4pm of May 22, 1991 up to 4pm May
22, 1992.
On June 13, 1992 one of the insured properties were razed by fire. Masagana
tendered, which UCPB accepted, 5 manager checks as renewal premium
payments of the policy. Masagana demanded for indemnification for the insured
property but UCPB rejected the claim on the ground that the policy was not
renewed, there was notice of non-renewal and the properties were burned
before the tender of premium payment. The Lower Court and CA decided in
favor of Masagana. According to the CA, the tender of payment had the effect
of renewing the policy on the basis of Policy Condition No.26 which provides
that unless there is notice of non-renewal, the insured shall be entitled to renew
the policy upon payment of premium. Both courts also found that Masagana
had been granted by UCPB a 60-80 day credit term for the renewal of the
policies. This had been the parties practice at the time the claims were filed. As
to the notice of non-renewal the CA found that Masagana received from
Ultramar Reinsurance Brokers (UCPBs reinsurance facility) on April 15, 1992, a
confirmation that only 67.5% would be reinsured. Hence, although the notice of
non-renewal was sent beyond the 45day period from the expiry date, UCPB still
accepted the premium payment and assumed the risk.
The case was elevated to the SC and in 1991, the SC ruled that the fire
insurance was not extended or renewed by implied credit arrangement.
Issue: W/N the insurance policy was extended by the payment of the premium.
Held: Yes. The following facts have been established: 1) the policies issued by
UCPB to Masagana have been annually renewed; 2) UCPB had been granting
Masagana 60-90day credit term to pay the premiums on the renewed policies;
3) no valid notice of non-renewal (no proof that notice was sent and received);
and 4) the premiums for the renewal were paid Masagana within the credit term
and were accepted by UCPB.
Section 77 of the Insurance Code came from Section 72 of the Insurance Act.
The Insurance Act expressly provided that parties are permitted to come to an
agreement to grant the insured credit extension of premium due. However, this
portion has not been restated in the Insurance Code. Nevertheless, there are
still exceptions to the mandate of Section77 (Insurance Code).
The 1st exception is provided by sec. 77 itself that is in case of a life or
industrial life policy whenever the grace period provision applies.
The 2nd exception is covered by sec. 78 which refers to the acknowledgment
in a policy of the receipt of premium as conclusive evidence of its payment and
therefore making the policy binding despite a stipulation that the policy it shall
not be binding until premiums are actually paid.
The 3rd exception was laid down in Makati Tuscany Condominium Corp vs CA
(one of the next few cases) wherein the SC ruled that sec. 77 may not apply if
the parties have agreed to the payment of installments of the premium and
partial payment has been made at the time of the loss. It appears in that case
the parties intended the policy to be effective despite the staggered payment of
premiums. The insurers acceptance of payment speaks loudly of its intention to
honor the policies it issued.
The SC also agreed with the CA ruling in the Tuscany case that sec. 77 merely
precludes the parties from stipulating that the policy is valid even if premiums
are not paid but does not expressly prohibit an agreement granting credit
extension. Hence, Tuscany provided a 4th exception that the insurer may grant
credit extension for the payment of the premium. This is also in consonance
with Art.1306 of the Civil Code binding the parties as to their agreement.
Estoppel also bars UCPB from using sec. 77 when Masagana relied in good
faith to their practice of granting credit extension.
In sum, Section 77 provides; An insurer is entitled to payment of the
premium as soon as the thing insured is exposed to the peril insured
against. Notwithstanding any agreement to the contrary, no policy or
contract of insurance issued by an insurance company is valid and
binding unless and until the premium thereof has been paid, except in the
case of a life or an industrial life policy whenever the grace period
provision applies.
The exceptions to this are:
1) Life or industrial life policy where the grace period applies (Section 77)

!"#$%&"'( * &++,- .$!/#0" 42
Anbochi, Atillo, Weigand
Taken from Rhys Alexeis reviewer. Digests from upperbatch.

2) Acknowledgement in the policy or contract of insurance of the receipt
of payment. (section 78, wherein acknowledgement if the conclusive
evidence of payment to make the policy binding notwithstanding a
proviso that it shall only be binding upon full payment.)
3) If the parties agreed to the payment in installments of the premium and
partial payment has been made at the time of the loss. (Makati
Tuscany v CA). The parties agreed or intended for the premium to be
in installments, and partial payment had already been made at the time
of the loss.
4) Insurer granted a credit extension for the payment of the premium.
(Makati Tuscany v CA). If the insurer has granted the insured a credit
term for the payment of the premium and the loss occurs before the
expiration of the term, recovery on the policy should be allowed even
though the premium is paid after the loss but within the credit term.
5) Estoppel (UCPB v Masagana Telemart). Where the insurer has
regularly allowed a credit term for the renewal of the policy. Even if it
did not grant a subsequent credit term, if the insured relied in good
faith on the practice of extending a credit term, estoppel is present.
6) Cover Notes. (section 52, no payment of the premium is yet made, as
it still cannot be computed, before the issuance of the policy.)
American Home v Chua
Facts: American Home is a domestic corporation engaged in the insurance
business. Respondent Chua obtained a fire insurance policy from them
covering the stock-in-trade of his business, Moonlight Enterprises. It was due to
expire on March 25, 1990.

On April 5, 1990, Chua issued a check to American Homes agent, James Uy,
as payment for the renewal of the policy. Uy delivered a Renewal Certificate to
Chua. The official receipt was issued on April 10. A new policy was issued
wherein the period of insurance was for the 1 yr. period beginning March 25,
1990. However, on April 6, 1990 (note, one day after Chua issued the check),
Moonlight Enterprises was completely razed by fire. Chua filed and insurance
claim with American Home & 4 other co-insurers. American Home refused to
honor the claim, forcing Chua to file a complaint.
American Home claimed that there was no existing insurance contract when the
fire occurred since Chua did not pay the premium. It invoked Sec. 77 of the
Insurance Code and cited the Arce case where the SC ruled that unless and
until the premium is paid, there is no insurance. It alleged that when the fire
occurred, the insurance contract was not yet subsisting pursuant to Art. 1249 of
the NCC, which recognizes that a check can only effect payment once it has
been cashed. The check could not have been cleared on April 6. In fact, the
Official Receipt was only issued 4 days after the fire.
The lower courts ruled in favor of Chua, holding that Chua paid by check the
day before the fire, which was acknowledged in the renewal certificate of the
agent.
Issue: W/N there was a valid payment of premium considering that the check
was cashed after the occurrence of the fire.
Held: Yes. The general rule in insurance laws is that unless the premium is paid
the insurance policy is not valid and binding. The only exceptions are life and
industrial life insurance. The RTC and CA found that there was a valid check
payment. The renewal certificate issued to Chua contained that
acknowledgment that premium had been paid.
Sec. 306 of the Insurance Code provides that any insurance company which
delivers a policy to an insurance agent shall be deemed to have authorized
such agent to receive on its behalf payment of any premium due. In this case,
the best evidence of such authority is the fact that American Home accepted the
check and issued the OR.
It is also bound by its agents acknowledgment of the receipt of payment. Sec.
78 states that an acknowledgment in a policy of the receipt of premium is
conclusive evidence of its payment. This section established a legal fiction of
payment and should be interpreted as an exception to Sec. 77.


Grepalife v CA
Facts: Cortez applied, through Grepalifes underwriter Mrs. Siega, a 20 year
endowment plan for PhP 30,000. After undergoing the requisite medical
examinations, the policy was issued to him. He was informed by Mrs. Siega that
he may pay the first premium within the 30 day grace period from the date of

!"#$%&"'( * &++,- .$!/#0" 43
Anbochi, Atillo, Weigand
Taken from Rhys Alexeis reviewer. Digests from upperbatch.

delivery. During this time, Cortez paid the first premium on installment.
However, subsequently, Grepalife informed Cortez that the policy was no longer
in force and asking Cortez to remit the balance to complete the first premium
and furthermore, asking him to undergo another medical examination at his own
expense. Aggrieved, Cortez went to court for rescission and damages.
Issue: W/N Cortez is entitled to the return of his premium
Held: Yes. The 3 installments made by Cortez were all made within the 30
grace period and were even acknowledged by Grepalife. Thus, when Grepalife
informed Cortez 4 MONTHS after it received the first premium that the policy
was never in force, and at that he must pay another premium and undergo
another medical examination to make the policy effective, it acted in gross bad
faith, and committed a serious breach of the contract of insurance. Because
Grepalife repudiated the contract rendering Cortez unprotected during the time
the latter falsely believed that he was insured, Grepalife must return whatever it
may have received. Grepalife should have informed Cortez of the deadline for
paying the first premium before or at least upon delivery of the policy to him, so
as to enable him to comply with what was needed, and would not have been
misled into believing that his life and his family were protected by the policy, hen
actually, they were not. And if the premium paid by Cortez was unacceptable for
being late, it was Grepalifes duty to return it. By accepting the premiums
without giving him the corresponding protection, the company acted in bad faith.
Section 79, 81 and 82 of the Insurance Code provides that when the insured is
entitled to the return of premium paid. Since Chuas policy was in fact
inoperative or ineffectual from the beginning, the company was never at risk,
hence, it is not entitled to keep the premiums paid.
Makati Tuscany v CA
Facts: In March 1982, American Home (private respondent), represented by
American International Underwriters Phils. issued an insurance policy in favor of
Makati Tuscany on its building, for a period until March 1983 with the total
premium of 466K. The premium was paid in 4 installments (March, May, June
and November), all were accepted by American.
American issued a new policy to Makati for a period of 1 year from March 1983.
Premiums were paid and accepted in 4 installments again. The policy was
renewed again to cover 1984. This time, Makati made 2 installments only, and
then stopped paying the balance. American sued. Makatis reason for
discontinuing payment was that it noticed in the receipt of payments the
reservation that says: acceptance of this payment shall not waive the insurers
rights to deny liability on any claim under the policy arising before such payment
or after expiration of credit clause; and that the insurer is subject to no loss prior
to the premium payment and if there is loss, such is not covered. Makati also
claimed that the policy was never binding and it asked for a counterclaim for all
premiums paid for 1984-1985.
The Trial court dismissed the case and the counterclaim. CA modified ordered
Makati to pay the rest of the instalments and denied its counterclaim.

Issues:
1) W/N the policy was valid;
2) W/N payment in installments was allowed; and
3) W/N Makati Tuscany must pay the balance.
Held:
Yes to all three. An insurance policy can be valid even if the premiums were
paid in installments. The facts show that both parties intended the policy to be
binding and effective notwithstanding the staggered payments. This bolstered
by the fact that the policy remained for 3 years, with this manner of installment
payment being made. That American accepted the payments shows its
intention to honor the policies. Moreover, it would be iniquitous and unfair if
American could be allowed to deny liability on the non-payment in full of the
premiums. Basic principles of equity and fairness would not allow the insurer to
continue collecting and accepting the premiums, although paid on installments,
and later deny liability on the lame excuse that the premiums were not prepaid
in full.
Sec 77 merely precludes the parties from stipulating that the policy is valid even
if premiums are not paid. It does not prohibit an agreement granting credit
extensions and other agreements not contrary to law, public order, public policy.
It appearing from the peculiar circumstances that the parties intended to make
the 3 insurance policies valid, effective and binding, petitioner may not be
allowed to renege on its obligation to pay the balance of the premium after
expiration of the whole term of the third policy. Where the risk is entire and
the contract is indivisible, the insured is not entitled to a refund of the
premiums paid if the insurer was exposed to the risk insured for any
period, however brief or momentary.

!"#$%&"'( * &++,- .$!/#0" 44
Anbochi, Atillo, Weigand
Taken from Rhys Alexeis reviewer. Digests from upperbatch.

Grepalife v CA (refer to previous case)

Вам также может понравиться